Сохранен 233
https://2ch.hk/spc/res/133400.html
24 декабря Архивач восстановлен после серьёзной аварии. К сожалению, значительная часть сохранённых изображений и видео была потеряна. Подробности случившегося. Мы призываем всех неравнодушных помочь нам с восстановлением утраченного контента!
Аноним 08/11/14 Суб 09:23:29 #1 №133400 
14154278097970.jpg
Здравствуйте. не закидывайте говном, но я у вас впервые.
Вчера посмотрел Интерстеллар. Понятно, что фильм абсолютно не научный, а есть лишь пища для размышлений о самой концепции человеческого существования и о прочем хуё-моё. Но фильм таки натолкнул на то, чтобы я у вас попытался разъяснить некоторые моменты. Сразу скажу - я лютый гуманитарий и с физикой дружу плохо. Помогите разобраться с данными вопросами:

1. Может ли наблюдатель выжить упав в черную дыру за горизонт событий? Любой наблюдатель.
Моя логика говорит мне, чо тот же человек, даже каким-либо образом будучи не разорванным в клочья, не сможет выжить ни секунды, ибо попав за горизонт событий в его организме должны прекратиться все процессы. Ибо как, например, сигнал нервный дойдет с его мозга к тем же глазам, если за горизонтом событий даже те же фотоны уже не подвластны себе и лишь устремляются к дыре? Человек же должен сразу умереть. Правильно?

2. В фильме существует планета, на которой, из-за близости к дыре, очень медленно течет время, относительно корабля на дааааальней орбите. Там называлась цифра "1 час = 7 лет". Какая должна юыть гравитация на планете? У меня такое ощущение, что просто пиздецовая, т.е. очень большая, при которой человек бы не выжил. Так ли это?

3. Вопрос не совсем относящийся к фильму.
существует теоретическое понимание, что при увеличении силы гравитации время замедляется, что проверено, вроде даже как, опытами.
Так вот, считается, что если создать очень крупное поле, то можно получить некий эффект машины времени в будущее. Т.е. для тебя время будет течь медленнее, чем для людей, например, на Земле. Так вот. Летая вокруг черной дыры мы можем добиться такого эффекта. Оставим в стороне вопрос какими техничекими средствами мы эт оосуществим, НО какая гравитация будет на самом корабле и не убьет ли она людей? Для достижения эффекта замедления времени гравитация должна оказывать влияние непосредственно на "человека", или может оказывать влияние на корабль, а что там внутри уже все равно?

Спасибо заранее) Не тральте плес
Аноним 08/11/14 Суб 12:36:10 #2 №133408 
Читни рассказик Нивена "Нейтронная звезда". Вопросов меньше будет.
Аноним 08/11/14 Суб 13:28:05 #3 №133411 
>существует теоретическое понимание, что при увеличении силы гравитации время замедляется, что проверено, вроде даже как, опытами.
Это нынешнее научное заблуждение из разряда "Солнце вращается вокруг Земли".
Аноним 08/11/14 Суб 13:43:31 #4 №133413 
>>133411
пруфики?
Аноним 08/11/14 Суб 13:46:24 #5 №133414 
>>133413
Ищу вот. Могу скинь тебе статью, но ниасилишь ведь. А сжато, чтобы каждый местный аутист въехал, хрен найдешь.
Аноним 08/11/14 Суб 13:56:51 #6 №133415 
>>133413
http://www.is-si.ru/esy.pdf
В общем, держи статью.
В любом случае, "ВРЁТИ" обеспечено.
Аноним 08/11/14 Суб 13:57:49 #7 №133416 
>>133411
Солнце вращается вокруг земли, тут понятно, мол относительность. Но с силой гравитации же по-другому получается.
Объясни подробнее или дай ссылку плиз
Аноним 08/11/14 Суб 13:58:18 #8 №133417 
>1. Может ли наблюдатель выжить упав в черную дыру?
Для наблюдателя вне черной дыры,падающий так и не упадёт.
Для самого дыронафта он достигнет сингулярности таки упадёт.
Т.е. произойдет хуйня.Но,что бы хуйни не произошло черную дыру просто "зацензурили".
>в его организме должны прекратиться все процессы.
Нит.
Вот нарисуй на листе бумаги человека.Затем скрути лист в трубку и засунь в свою черную дыру.Вот так и там,пространство искривляется,а человек ,как бы нарисован на нем,и ему похуй.как бы.
Аноним 08/11/14 Суб 13:58:56 #9 №133418 
>>133415
Читаем. Спасибо.

А что насчет других вопросов в оп-посте
Аноним 08/11/14 Суб 14:01:57 #10 №133419 
>>133417
Почему для наблюдателя внешнего он не упадет?

Искоевление пространства. Понятно. Но на сам человеческий организм такие гравитационные силы разве не окажут смертельного воздействия
Аноним 08/11/14 Суб 14:03:22 #11 №133420 
Раз уж ОП-хуй задал такие вопросы то и я задам. Как будет выглядеть "замедление" времени около ЧД? Будет слоу мо или всё будет идти нормально, но "1 час = 7 лет"? Как будет выглядеть "ускорение" времени во время релятивистского полёта(на 0.99с скажем)? Всё будет убер быстро или нормально но "1 год = 1 секунда"?
Аноним 08/11/14 Суб 14:07:26 #12 №133421 
>>133420
Пойми прежде всего, что сам фильм - фантастический.
Тут вон учёные спорят, существуют ли вообще чёрные дыры (http://www.astronomy.ru/forum/index.php/topic,122549.0.html), а ты спрашиваешь о таких вопросах.
Аноним 08/11/14 Суб 14:10:35 #13 №133423 
>>133420
Кхм. Замедление времени на орбите ЧД вызвано тем, что орбитальная скорость близится к субсветовой. Никакого такого замедления времени от повышенной гравитации нет.
Аноним 08/11/14 Суб 14:37:00 #14 №133434 
>>133415
>http://www.is-si.ru/
Сеошник хуже червя-пидора.
Аноним 08/11/14 Суб 14:38:46 #15 №133435 
>>133415
>"ВРЁТИ"
/По/рашник, please.
>>133417
Студентик, пожалуйста.
>>133423
Ебать дебил.
Аноним 08/11/14 Суб 14:48:50 #16 №133438 
>>133419
>Почему для наблюдателя внешнего он не упадет?
Он будет замедляться по мере падения.Будет стремиться упасть.Это как 0.(9)99999999 стремится стать 1.Вот,вроде точно не станет же,а стремится.

>человеческий организм смертельное воздействие.
Да хуй знает.Должно раздавить.А может нет.Цензура же.Даже подумать об этом толком не возможно.Если по формулам то раздавит.
Аноним 08/11/14 Суб 15:13:05 #17 №133439 
>>133438
Но почему тогда существует сам горизонт событи и он "черный" а не состоит из "изображений" всего чт в него падает?
Аноним 08/11/14 Суб 16:04:59 #18 №133443 
>>133438
Алс ты ммеешь ввиду само тел или его "вмд" для наблюдателя?
Аноним 08/11/14 Суб 16:48:09 #19 №133449 
Раз уж тут тред "интерстеллара", еще вопрос вам:
Какова вероятность планеты на более-менее стабильной орбите вокруг ЧД?
Аноним 08/11/14 Суб 16:52:48 #20 №133450 
Возможно черная дыра не ,там, в космосе,а в нашем сознании.Такое слепое пятно.
Из своего 3х мерного пространства,общего для всех, с линейным временем,мы не можем даже допустить , что событие произойдет и не произойдет ни когда.

Бля,возможно мы и сами,со всей нашей ёбаной вселенной медленно стекаем по горизонту событий, какой нибудь черной дыры.
по этому богушка не может найти нас
Аноним 08/11/14 Суб 16:56:03 #21 №133453 
>>133449
ЧД есть почти во всех центрах галактик, а наша планета на более-менее стабильной орбите относительно центра.
Аноним 08/11/14 Суб 17:14:02 #22 №133460 
>>133400
> должен сразу умереть.
Для "щастливчега" оставшегося снаружи, перый будет умирать долго-предолго.
Но сам щаслифчик сдохнет довольно быстро, наблюдая при этом закат Вселенной.
Аноним 08/11/14 Суб 17:30:24 #23 №133465 
>>133415
Прочитал до вероятных оппонентов. Очень интересно. Особенно где про противоречме дефекта массы и замедления времени, при котором 1кг "уменьшается", а секунда " увеличивается".
Аноним 08/11/14 Суб 21:21:12 #24 №133493 
14154708727190.jpg
>>133450
>Бля,возможно мы и сами,со всей нашей ёбаной вселенной медленно стекаем по горизонту событий, какой нибудь черной дыры.

Аноним 08/11/14 Суб 22:25:31 #25 №133495 
>>Может ли наблюдатель выжить упав в черную дыру за горизонт событий?
Современная физика не очень хорошо понимает, что происходит с веществом внутри черной дыры. Так что все может быть. может там внутри розовые единороги
>>Ибо как, например, сигнал нервный дойдет с его мозга к тем же глазам, если за горизонтом событий даже те же фотоны уже не подвластны себе и лишь устремляются к дыре? Человек же должен сразу умереть. Правильно?
Нет. Горизонт событий это просто такая математическая абстракция, в нее нельзя врезаться. Причем она зависит от наблюдателя.
Проблемы будут в основном из-за:
1. Приливных сил. Черная дыра звездной массы просто разорвет на куски объект размером с человека.
2. Джетов, излучения. Просто поджарит издалека.
>>У меня такое ощущение, что просто пиздецовая, т.е. очень большая, при которой человек бы не выжил. Так ли это?
Астронавты на МКС не чувствуют смлу тяжести. Теперь замени землю на черную дыру а МКС на ту планету.
Аноним 09/11/14 Вск 10:04:08 #26 №133522 
>>133493

Да-да, и наше солнце это черная дыра, а "пятна" на солнце появляются тогда, когда аккреция от которой солнце и "горит" рассеивается. И жизнь на земляшке в виде людей, сов и прочих тварей - всего то проявление аккреции воды в очень замедленном времени.

Ебануться можно.
Аноним 09/11/14 Вск 10:28:55 #27 №133524 
>>133495
>> Так что все может быть. может там внутри розовые единороги
Я думал вс епроще - тупо черная сфера из-за гигантской массы, находящейся в центре. Черная, потому что свет не вылетает. Думал все вот так просто. Какие единороги...

>>133495
>Нет. Горизонт событий это просто такая математическая абстракция, в нее нельзя врезаться.
Ну не врезаться, а "пройти" сквозь эту границу.
Вот. Приливные силы и излучения. да.
>>133495
>Астронавты на МКС не чувствуют смлу тяжести. Теперь замени землю на черную дыру а МКС на ту планету.
Ну ты сравнил тоже.) В любом случае, то есть сила тяжести будет действовать на корабль, н оне на людей, что внутри?
При каком раскладе можно было бы получить эффект замедления времени? Должны ли люди чувствовать изменение силы тяжести?

Аноним 09/11/14 Вск 12:29:40 #28 №133536 
>>133449
Ну естьнаверное сверхмассивные дыры, вокруг которых теоретически может вращаться планетарное тело довольно близко к горизонту событий. Но вот на расстоянии "ололо замедление времени 1 час=7 лет" уже ничего, сформированного собственно гравитацией вращаться не может, ибо распидорасит приливными силами.
Аноним 09/11/14 Вск 12:42:20 #29 №133539 
>>133524
>сила тяжести будет действовать на корабль, но не на людей, что внутри?
Нет, для круговой орбиты необходима первая космическая. Для Земли она ~равна 7.9 км/с. На такой скорости центробежная сила нивелирует действие гравитации. То есть корабль и экипаж будет ощущать силу тяжести в полной мере, но эта сила будет компенсироваться центробежной и общий вектор будет практически равен нулю. Другое дело, что первая космическая на низкой орбите для ЧД будет иметь релятивистские значения. И да, за счет такой скорости эффект лоренцевого замедления будет иметь место. Но, скорее всего, не для корабля и экипажа, а для нуклонов, из которых они когда-то состояли. Ибо на таких скоростях и значениях гравитации приливные силы, вероятно, будут губительны даже в столь малых масштабах.
Аноним 09/11/14 Вск 13:33:49 #30 №133542 
>>133524
>При каком раскладе можно было бы получить эффект замедления времени?

Да даже на МКС есть.Не такой пиздецовый, что бы стать основой сценария,но измеряемый.

> Должны ли люди чувствовать изменение силы тяжести?
Не понял.От гравитации в космосе?Нет.
>>133536
вращаться не может, ибо распидорасит приливными силами.
Не факт.Не факт ,что чд это сколлапсировавшие звёзды.
sageАноним 09/11/14 Вск 14:05:12 #31 №133543 
>>133522
И наше солнце это черная дыра,
А "пятна" появляются тогда,
Когда аккреция рассеивается...

И на земляшке жизнь в виде людей,
И сов и прочих тварей,
Всего то проявление аккреции...
Аноним 09/11/14 Вск 14:14:30 #32 №133546 
>>133542
>вращаться не может
Я про сверхмассивные дыры писал, приливные силы у таких на горизонте событий не так уж и фатальны, другое дело, что орбитальная скорость должна быть очень высокой.
Аноним 09/11/14 Вск 15:45:19 #33 №133551 
>>133539
Точно! Там же центробежная уравнивает. Понял. Спасибо!
Аноним 09/11/14 Вск 19:52:28 #34 №133582 
>>133543

Охуенно :3
sageАноним 09/11/14 Вск 20:33:26 #35 №133595 
>>133400
>Понятно, что фильм абсолютно не научный
Иди на хуй.
Аноним 09/11/14 Вск 20:47:44 #36 №133597 
>>133595
Почему?
sageАноним 09/11/14 Вск 20:54:51 #37 №133600 
>>133597
Потому что Кип Торн.
Аноним 09/11/14 Вск 21:05:37 #38 №133602 
>>133600
ну и?
sageАноним 09/11/14 Вск 21:07:49 #39 №133603 
>>133602
Хуи.
Аноним 09/11/14 Вск 21:13:54 #40 №133605 
14155568343200.gif
>>133603
>>133603
sageАноним 10/11/14 Пнд 02:38:53 #41 №133629 
>>133536
>Я про сверхмассивные дыры писал, приливные силы у таких на горизонте событий не так уж и фатальны.
НУ.
>орбитальная скорость должна быть очень высокой
Нахуя?
писька уверен ,что чд ,это
коллапсир.
Я думаю ,что она - пространство.
азаза интерскалар нет нет нет нет нет
Аноним 10/11/14 Пнд 09:38:04 #42 №133645 
14156014842600.jpg
А разве человека не распидорасит в фарш еще при приближении к ЧД из-за приливных сил?
Аноним 10/11/14 Пнд 15:55:26 #43 №133662 
14156241268290.png
>>133645
Не обязательно.
Аноним 10/11/14 Пнд 19:43:23 #44 №133686 
>>133400
>фотоны уже не подвластны себе и лишь устремляются к дыре?

Горизонт событий это и есть дыра.

>У меня такое ощущение, что просто пиздецовая, т.е. очень большая, при которой человек бы не выжил. Так ли это?

Нет. Причём тут г8равитация на самой планете? Замедление времени идёт за счёт искривления пространства-времени вблизи дыры.

>НО какая гравитация будет на самом корабле и не убьет ли она людей?

Точно гуманитарий. Гравитация компенсируется центробежкой, как и на любой орбите, земной нпр.

>Не тральте плес

Да тут ты скорее тралишь.
Аноним 11/11/14 Втр 00:20:34 #45 №133747 
>>133645
при сверхмассивной ЧД вроде как нет.
Поэтому "полет" Куппера не столь уж антинаучен.
Аноним 11/11/14 Втр 00:22:35 #46 №133748 
Слушайте, а посоветуйте что почитать чтобы базовых знаний по космосу набраться, или типа того. А то вы порой тут такое в тредах обсуждаете и мне знаний физики с института не хватает.
Где-то посоветовали для начала можно хокинга читануть. Дайте советов.
Аноним 11/11/14 Втр 00:23:36 #47 №133749 
>>133748
Кип Торн - Черные дыры. Довольно легко читается
Аноним 11/11/14 Втр 00:26:21 #48 №133752 
Слушайте, я всегда думал что вот летишь ты такой, время замедляется, и мне это представляется типа растягивание, а когда ты возвращаешься обратно, то время сжимается обратно в норму, те как бы ты нагоняешь то когда был в замедлении. Ну те для стороннего наблюдателя ничего не изменится и ты вовсе не прилетишь молодым, когда все уже постарели.
Аноним 11/11/14 Втр 00:28:26 #49 №133753 
>>133749
Мне бы что-то такое общее, чтобы и черные дыры и скорости времени, вся эта относительность и тд, чтобы база была, а потом можно будет уже углубляться в интересующие темы.
Аноним 11/11/14 Втр 00:29:57 #50 №133754 
>>133753
Окей. тогда Хокинг- Краткая история времени
Аноним 11/11/14 Втр 00:33:19 #51 №133755 
>>133754
Ясн, спасибо.
Я наверно и остальные книги у него зацеплю, вроде все говорят, что норм. Там же нет особого порядка в котором читать.
Аноним 11/11/14 Втр 15:07:29 #52 №133792 
>>133400
>Понятно, что фильм абсолютно не научный
>Сразу скажу - я лютый гуманитарий и с физикой дружу плохо
Ты долбоеб?
Аноним 11/11/14 Втр 15:16:13 #53 №133793 
>>133792
Да, это вполне в гуманитария-ебаната заявить свое безапелляционное авторитетное мнение в той области, где он ни сном, ни духом.
Аноним 11/11/14 Втр 17:27:35 #54 №133801 
>>133747
>пролетел сквозь
>Вынырнул на другом конце галактики и так несколько раз
>не столь уж антинаучен
Не, я кино не смотрел так-то, может все по другому было.
Аноним 11/11/14 Втр 17:30:14 #55 №133802 
>>133801
Никто не знает, что там в этой ебучей сингулярности, может там вообще мультики с понями показывают, так что глюки Куппера внутри ЧД могут быть любыми.
Аноним 11/11/14 Втр 17:30:34 #56 №133803 
>>133752
Ну и что, что ты думал? Лоренцево замедление доказано экспериментально, и соответствующие поправки вводятся во внутренние часы спутников, никаких "нагоняний" не случается.
Аноним 11/11/14 Втр 18:12:37 #57 №133809 
Аноны, поясните такой момент: в умных книгахъ пишут, что в сильных гравитационных полях течение времени для стороннего наблюдателя замедляется, т.е. если некий космонавт начнёт падать в ЧД, то сторонний наблюдатель будет видеть, как он всё больше и больше замедляется по мере приближения к горизонту событий, но так никогда его и не пересечёт, в то время как для него окружающая Вселенная будет ускоряться всё больше и больше пока не случиться тепловая смерть Вселенной или еще какой БП. Из всего этого вытекает вопрос: а как же с аккрецией вещества на черную дыру? Вещество же разгоняется, падает на неё, из-за этого образуется рентгеновское и туча другого излучения. Разве материя не должна так же замедлиться и никогда на ЧД не упасть?
Аноним 11/11/14 Втр 18:41:33 #58 №133814 
>>133802
>глюки Куппера внутри ЧД могут быть любыми
Ну охуеть давай теперь всё фэнтези научным назовем. Вдруг все описываемое Толкиеном происходит под горизонтом событий черной дыры. Никто ж не знает, что там. Вполне научно значит.
Аноним 11/11/14 Втр 19:11:28 #59 №133819 
>>133809
>пока не случиться тепловая смерть Вселенной или еще какой БП
Очередная охуенная сказка для легковерных. Я вообще тоже глубже науч-попа не знаток, но очевидно, что падение на черную дыру - пусть и долгий, но таки конечный процесс. И получается, что теоретический космонавт будет наблюдать ускорение времени, но чтобы увидеть какие-то макропроцессы вселенной, свет от этих процессов в первую очередь должен дойти до него. Блять, вообще это очень непростой вопрос, для полноценного рассмотрения которого придется привлечь несколько теорий, некоторые из которых мне даже не известны. Проще говоря падение за горизонт - процесс конечный. Пусть допустим пару тысячелетий. Гравитация чд, допустим, замедляет время падающего в миллиард раз. Значит весь процесс падения он увидит за 200031^6/10^9=62 секунды. И это при условии гипотетического сферического не разрываемого космонавта. Ну и вообще чем ближе к горизонту, тем меньше света доходит до глаза и тем меньше площадь обзора. На самом краю космонавт увидит лишь маленькую точку внешнего пространства, окруженного абсолютной темнотой. Не помню, как это объясняется, но это так.
>Разве материя не должна так же замедлиться и никогда на ЧД не упасть?
Замедлится время для конкретно движущегося объекта. То есть какой-нибудь пи-мезон с временем жизни 10^-23c проживет 10^-14c.
Аноним 11/11/14 Втр 19:41:43 #60 №133823 
>>133809
Для начала пойми, что то, что видит наблюдатель, отличается от того, что видит сам объект. Для объекта падение в ЧД процесс конечный, и более того, незаметный — понять, пересёк ли он границу дыры, он может только по наблюдению за окружающими объектами, для него же самого ничего не изменилось, падает и падает. И очень скоро упадёт (куда точно и в каком состоянии там находится вещество — неизвестно, есть много теорий о внутреннем строении ЧД).
У тебя проблема с пониманием искажения времени в релятивистских процессах, оно не абсолютно, если ты видишь, что объект замедлился у границы ЧД, это не значит, что он реально замедлился.
Аноним 11/11/14 Втр 19:46:20 #61 №133824 
>>133747
Там антинаучно многое другое.
Прям совсем многое
Аноним 11/11/14 Втр 19:48:34 #62 №133825 
>>133748
>мне знаний физики с института не хватает
Лол, институтские знания - хуита.

А вообще читай Владимира Сурдина "Астрономия 21 век", это не для начинающих, но всё объясняется очень круто и формул вообще почти нет. Крутая крч книжка. А что непонятное там встретишь - гугли.
Аноним 11/11/14 Втр 19:50:55 #63 №133826 
>>133752
Для тебя время не замедляется. Суки, чё ж вы элементарные вещи из ТО понять не можете? При увеличении скорости/массы время замедляется относительно(!) других, удалённых объектов, а не само по себе. Главное слово и главный принцип тут - ОТНОСИТЕЛЬНО чего-то. Хотя ты сам, лично, вообще ничего не заметишь и не почувствуешь.
Аноним 11/11/14 Втр 19:55:39 #64 №133827 
Вообще там много косяков действительно антинаучных почему ЧД светится? Какого хуя? Откуда по ней плазма растекается? Вообще же нихуя не понятно. Точнее понятно - такое может быть при условии ТДС (тесной двойной системы), т.е. у ЧД должна быть звезда-спутник, у которой ЧД берёт материю. Купер там еще упоминал какую-то нейтронную звезду, но это вообще хуй пойми о чём он., но все они сделаны лишь в угоду сюжету, иначе если их исправлять - и сюжет весь развалится же.
Аноним 11/11/14 Втр 19:57:31 #65 №133830 
>>133819
>Очередная охуенная сказка для легковерных.

Ска, я написал про тепловую смерть просто в качестве примера. Хотя у меня нет причин считать, что тепловая смерть Вселенной невозможна.
Аноним 11/11/14 Втр 20:08:13 #66 №133836 
>>133793
Вы ебантяи, а кака научность в полетах космонавта в ЧД, в библиотеке нахуй в ЧД и в путешествии в прошлое с помощью гравитации? Тут можно быть хоть хуймунитарием, чтобы понять фантастичгость

Оп
Аноним 11/11/14 Втр 20:11:45 #67 №133840 
>>133836
Ну насчёт 5-ти измерений ну не в черной дыре, конешно же не всё так фантастично тащемта. Хотя и непонятно почему пребывание в пятимерном пространстве его так ограничивало, что он тупо не мог войти к ней в комнату и вломить пиздюлей.
Аноним 11/11/14 Втр 20:22:35 #68 №133844 
14157265557250.jpg
>>133840
Я не про пятимерное пространство, я про библиотеку в черной дыре.


Алсо я так и не понял ответа на вопроч, почему человека сразу не распидорасит от приближения к ЧД. Почему фотоны засасывает, а человек даже не амечает переход через горизонт событий?
Объясните кто-нибудь. В оп-посте я жтот же вопрос задаво чуть подробн
Аноним 11/11/14 Втр 20:23:32 #69 №133845 
>>133819
Еще один гуманитарий, откуда вы лезете?
>Пусть допустим пару тысячелетий
>Гравитация чд, допустим, замедляет время падающего в миллиард раз.
Ебнется он туда, на раз два. Никаких тысячелетий не нужно. Бесконечно долго он падать будет только для стороннего наблюдателя.
>для полноценного рассмотрения которого придется привлечь несколько теорий, некоторые из которых мне даже не известны
ОТО вполне достаточно.
>>133824
>Там антинаучно многое другое.
Что именно, или ты очередной "Я нихуя не разбираюсь, но мнение имею"?
>почему ЧД светится?
Оно не светится, светится вещество, падающее на ЧД.
>Откуда по ней плазма растекается?
Штоу?
>т.е. у ЧД должна быть звезда-спутник, у которой ЧД берёт материю
Она может брать материю откуда угодно, мимокрокодильного облака газа, например, или планеты.
>Купер там еще упоминал какую-то нейтронную звезду
Может быть, что это второй, третий, четвертый компонент, есть звездные системы в несколько компонентов, никаких противоречий не вижу.
>и сюжет весь развалится же.
Сюжет там вообще не об этом и не про это.
>>133836
>Вы ебантяи
Он, наш петух-гуманитарий закукарекал, заместо Авроры.
>в библиотеке нахуй в ЧД и в путешествии в прошлое с помощью гравитации?
Он имел возможность гравитационно влиять на небольшой участок пространства в любой момент времени. По сути, пребывая в пяти измерениях он мог влиять на четырехмерный мир, не имея возможности быть его частью, точно так же, как ты не можешь существовать в одномерном мире.
>а кака научность в полетах космонавта в ЧД
Такая, что в теории есть ЧД, в которые можно влететь, без вреда для здоровья.
>и в путешествии в прошлое с помощью гравитации?
Вот про это столько понаписано литературы, где доступным даже дебилу языком объяснено, как с помощью гравитации можно путешествовать во времени. Ах, да. Ты же гуманитарий, кроме художественной литературы тебе ничего не доступно.
Аноним 11/11/14 Втр 20:27:51 #70 №133848 
>>133845
Ну и расскажи мне пожалуйста про путешествия В ПРОШЛОЕ с помощью хоть гравитации хоть твоей мамки. Почему я, гуманитарий, понимаю невозможность этого, а ты, умник, нет?

Ps пятимерные измеоения, телрии блять... Наука епты. Ахуеть теперь
Аноним 11/11/14 Втр 20:46:13 #71 №133852 
>>133848
Смотрите, говно говорящее не имея никаких базовых знаний, не может купить себе книгу и прочитать 300 страниц, оно желает, что бы ему тут разжевали в двух постах. Про измерений, больше 4-х замечательно написано в книгах по теории суперструн, и в работах Яу. Ах да, опять книжки читать, не про эльфов.
Аноним 11/11/14 Втр 20:48:15 #72 №133853 
>>133852
Ты жьпьй не верти со своиси изсеренмями.
Я тебя спрашиваю - поясни за мутешествия в прошлое или иди будь обоссан, мудила
Аноним 11/11/14 Втр 21:03:38 #73 №133856 
14157290180770.jpg
>>133852
Вот, например, если бы главный герой фильма не передал себе бы координаты базы наса из ЧД, то чт бы произошло? Все его нахождение на этой миссии неожиданно стерлось бы, а может открылась бы параллельная вселенная? Расскажи гуманитармю, технарь ебаный. Такую херню можно найти в любом примере. Так расскажи мне что делать с этим противоречмес
Аноним 11/11/14 Втр 21:13:01 #74 №133857 
14157295817630.png
>>133844
Ну смотри, приливные силы. Картинка в википедии достаточно наглядна, но я нарисую тебе сам. Вот ноги васи чуть ближе к дыре, следовательно на них гравитация действует сильнее, ибо чем меньше расстояние, тем больше сила. Собственно такой формат присущ для любого тела, даже для Земли. Почему мы на Земле не рвемся на куски. Тут появляется понятие градиент гравитации. Градиент в конкретном случае - значение, на которое увеличивается сила при приближении к притягивающему объекту. На Земле он очень мал, у меня даже был курс гравиметрии, но точное значение я не вспомню, что-то вроде 0,23 мГал/м. мГал= 0,001 Гал. 1 Гал= 1см/c^2, то есть где-то 1/1000 от ускорения g. На величину этого градиента влияет компактность массы. Так вот ЧД - наиболее компактный по массе тип объекта. Поэтому градиент будет огромен и ноги Васи будут притягиваться со значительно большей силой, нежели голова. Поэтому тело Васи начнет испытывать растяжение, причем такое, что кровь кишки. А кровь кишки на атомы. Ну вообще сразу на атомы, но для кровавого словца. И это при условии, что Вася будет двигаться по орбите. Если Васю телепортировать к горизонту событий с нулевой скоростью по отношению к ЧД, то гравитация разорвет Васю сама по себе, ибо Вася испытает многомиллиардную перегрузку.
Но в нынешней теории о ЧД появились сверхмассивные объекты, масса которых настолько велика, что горизонт событий находится очень далеко от центра. То есть не возможно современными методами создать теорию, заглядывающую под горизонт, но экстраполяция нынешних знаний предсказывает, что расстояние от горизонта события до основного тела СМЧД очень велико. И по этой причине градиент будет достаточно мал, чтобы не разрушать молекулярные связи. Тут всплывают разные понятия. Гравитация и скорость убегания. Скорость убегания - вторая космическая. Вот та граница, на которой она превышает с и есть горизонт событий. Градиент же не имеет постоянной величины для конкретного объекта и на сильном удалении может иметь нелетальные значения в массштабах человеческого тела и даже космического аппарата. Следовательно в случае со СМЧД можно вращаться по орбите предельно близко к горизонту событий на околосветовой скорости. В состоянии покоя относительно СМЧД находится нельзя, миллиардные перегрузки никто не отменял. Но нелетально вращаться теоретически можно. Теоретически даже можно нелетально погрузится под горизонт событий, но вот что там будет сказать нельзя. Физической модели для сингулярности пока нет.
Аноним 11/11/14 Втр 21:24:34 #75 №133859 
>>133845
>Ебнется он туда, на раз два. Никаких тысячелетий не нужно.
Лол, шурупы на заводе микроскопом заколачиваешь, технарь? "На раз два" - охуенный ответ. Для точного ответа нужно множество параметров, а 2000 лет приведены были для наглядности, потому что сравнение 2000 лет и 60 секунд человеку более просто дается, нежели сравнение наносекунд и фемтосекунд.
Аноним 11/11/14 Втр 21:25:16 #76 №133860 
>>133856
Никто тебе этого не расскажет, ибо:
1. Мы не знаем как устроено время в принципе, идёт ли оно одним потоком, где можно перемещаться только вперёд (в будущее) или же время имеет бесконечное множество параллельных потоков, которые могут сообщаться. Отсюдова и все эти ваши эффекты бабочки, парадоксы неродившегося дедушки и иже с ними. А может и времени вообще как такового не существует и оно воспринимается нами лишь как субъективная сущность/математическая абстракция/что-то другое.
2. Беря во внимание п.1 задаваться такими вопросами о ситуации в фильме глупо, ибо ситуация на данный момент гипотетическая и не имеющая ничего общего с реальностью.
Аноним 11/11/14 Втр 21:32:16 #77 №133863 
>>133845
Блять, забыл.
>ОТО вполне достаточно.
Ну да, теоретическое обоснование ЧД сейчас сталкивает практически все теории макромира и микромира, а тебе ОТО достаточно. Похуй, что у нас микрочастицы на сверхсветовых скоростях, тебе ОТО достаточно.
Аноним 11/11/14 Втр 21:41:53 #78 №133864 
>>133857
Вот я про то же!
Не слышал в жизни такого понятия как "градиент гравитации" чисто умозрительно я представлял, что тело человека должно же пиздецово вытянуться, при приближении к дыре и ГРОБ ГРОБ и далее по тексту.
А насчет второй части... То есть гравитация будет ОСНЕ сильной, но из-за малого градиента человека не разорвет, а просто начнет нехило разгонять по направлению к r=0? Спасибо тебе, адекват. Вроде я понял тебя.
>>133860
Ой, серьезно, не скажет? Так же как и не скажет каким цветом у бога волосы, потому что мы не знаем "как устроен бог"?
PS если ты не тот человек, с кем я вел дискуссию, то сори за грубость.
Аноним 11/11/14 Втр 21:45:15 #79 №133865 
>>133864
<...>НО чисто умозрительно я представлял <...> -фикс
Аноним 11/11/14 Втр 21:45:33 #80 №133866 
>>133853
>>133856
>Я тебя спрашиваю - поясни за мутешествия в прошлое или иди будь обоссан, мудила
Ты скучный и глупый.Вот смотри:
проекция сферы-круг.Мы можем это понять,так как видим в 3x измерениях.
Но проекцией чего является сфера?а?
Фильма хочет сказать:может быть,наше восприятие пространства и времени ограничено?Может мы видим только проекцию,только срез времени,только слой целого?
>не передал себе бы координаты базы наса из ЧД, то чт бы произошло?
Блять,этого не могло произойти.Все есть так как есть.Время для нас линейно.Но если это проекция,то оно может быть чем то другим.
>Так расскажи мне что делать с этим противоречмес
Страдай хули.



Аноним 11/11/14 Втр 21:50:45 #81 №133867 
>>133852
Мальчик, ты почитал науч-попа и решил, что стал умнее гуманитариев? Так перечитай, у тебя такая каша в голове, просто пиздец.
Аноним 11/11/14 Втр 21:56:06 #82 №133868 
14157321667300.jpg
>>133866
Всмысле не могло? Вот взял он и пока ревел в этой библиотеке не передал. Может не захотел, может забыл.

Какие блять сферы какие проекции... Опять жопой вертишь. Я тебе говорю пояснить за путешествия в прошлое, а ты на явное противоречие отвечаешь "его не случится" ахах ну тв и долбоеб... Технарь блять недоделаннвый
Аноним 11/11/14 Втр 22:01:12 #83 №133869 
14157324725510.jpg
Алсо за "ограниченность восприятия" и про прочие "возможно какие-т существа видят время, как мы "видим" высоту или длину" намного интереснее и без такого паыоса написано у Воннегута в романах. В частности в Бойне номер пять.
Аноним 11/11/14 Втр 22:04:38 #84 №133870 
>>133853
Ты сам себя уже обоссал:
>фильм абсолютно не научный
>я лютый гуманитарий

Учитывая, что научным консультантом фильма был Кип Торн, ты даже не в состоянии представить, масштабы своего обосрамса, ебанашка. И не забывай, ты всего-лишь никчемный кусок говна, который даже пожалеть не хочется.

Машина времени Готта, например, одно из возможных описаний путешествий во времени. Визуализация книжных полок с символично натянутыми струнами очень толсто намекает именно на этот вариант.
Гравитационное поле одиночной космической струны представляет из себя плоское пространство с клинообразным вырезом, вершина которого лежит на на струне, если двигаться вокруг струны, то расстояние будет меньше, хотя время будет не затронуто, а вот от если пустить вторую струну относительно первой, да еще с околосветовой скоростью, то клинья каждой из них сократят и пространственные и временные интервалы. Так как при таких раскладах появляются замкнутые времяподобные кривые, что означает - пиздуй-ка ты, Вася, в прошлое и не выебывайся.
>>133859
Будет оно падать, по обычному уравнению ускорения свободного падения, откуда ты взял свои сраные тысячелетия объясни сам. Высрал их головы, видимо.
>>133856
>не передал себе бы координаты базы наса из ЧД, то чт бы произошло?
Ничего, он бы никуда не полетел бы и ничего не было. В другом варианте, если нет возможности менять прошлое, то он бы не смог их НЕ передать. Тут как раз вся мякота и заключается, что путешествовать во времени, в теории можно, а вот менять что-либо - еще хуй знает.
> а может открылась бы параллельная вселенная?
По теории Мультивселенной, которая тоже толсто намекается в гиперкубе, который нашелся в сингулярности, все случилось и он передал, и не передал, передал, но его дочь оказалась тупой овцой и ничего не поняла, хотел передать, но часы сломались, передал, но брат уебал ей и в щщи у и она скопытилась, вообщем произошло все, что могло произойти, просто в разных вселенных были разные варианты, а он выбрал тот самый, ему нужный.
>>133863
>ЧД сейчас сталкивает практически все теории макромира и микромира
Падение макрообъекта, в виде космонавта в корабле в ЧД никаких проблем с микромиром не имеет. Так что сасай, лалка и разберись уже в тонкостях вопроса, ты подаешь надежды.
Аноним 11/11/14 Втр 22:08:52 #85 №133871 
>>133867
Мальчика в зеркале поищи, дебила кусок. Гуманитарии не могут в физические теории и прибегают к дешевым приемам демагогии, когда чувствуют, что отсасывают на всех фронтах. Конкретно тебе сказать по сути предмета нечего, знаний у тебя нет, но пиздануть что-нибудь чешется, поэтому ты пытаешься копротивляться, обвиняя оппонента в незрелости. Дешевый приемчик, гуманитарная ты шалава.
Аноним 11/11/14 Втр 22:10:02 #86 №133872 
14157330022420.jpg
>>Блять,этого не могло произойти.Все есть так как есть.Время для нас линейно.Но если это проекция,то оно может быть чем то другим.
Как писал мой любимый Воннегут "все что есть - был всегда, а вс что будет всегда существовало."
То есть ты утверждаешь, что "все есть так как есть" и у тебя небыло даже мизерного шанса вырваться из этой линейности и стать нормальным человеком, а не тем ебланом, коим ты являешься? Ну что ж, будем надеяться что по ходу этой линейности тебе уготовлено хоть какое-то повышение способностей в логике и в ведении диалога
Аноним 11/11/14 Втр 22:10:27 #87 №133874 
>>133868
Я только >>133866 написал.
>Всмысле не могло?
Без смысла,блять.Не произошло значит не могло.Хули бы ему не произойти,если оно могло.
>Я тебе говорю пояснить за путешествия в прошлое
Т.е. с будущим у нас проблем нет,я понимаю.

Аноним 11/11/14 Втр 22:11:10 #88 №133875 
>>133864
>а просто начнет нехило разгонять по направлению к r=0
Однозначно на него будет действовать сила притяжения. Начнет ли его разгонять - другой вопрос. Если не будет уравнивающих сил, то начнет, но настолько сильно, что вышеупомянутые миллиардные перегрузки. Такие значения будут равносильны разрыву приливными силами. В качестве уравнивающих сил может быть йоба-движок. Но вообще не может, ибо создает вектор тяги лишь в определенных точках аппарата. Поэтому центробежная сила так хорошо подходит, потому что действует на объект в целом и на каждую, составляющую его частицу в частности. Вообще все это - экстраполяция наших нынешних знаний, которая если и не совсем ошибочна, то точно не полна. О теоретических квантовых эффектах возле горизонта событий остается только гадать.
Аноним 11/11/14 Втр 22:19:55 #89 №133877 
14157335960030.jpg
>>133870
В чем я себя обосрал? В том ят я гуманитарий? Пощади...
Поебать кт там консультант. Фмльм вышел таким каков он есть - философской притчей, далёкой от науки как ты далек от защиты диплома, первокурс ебаный.

Отлично выкрутлся. Мулььивселенные. Точно. Красава. Я то думал... А у нас прост мультивселенные. Спасибо. Расходимся.


>>133874
Ну вот я завтра снмму интерстеллар, н в концовке оине передаст координаты. Что мне посоветует
Кип Торн для разрешения этой проблемы? А с будущим... Ну вот вокруг той же СМЧД полетать, время пойдет у нас медленнее и блищнец летавший вокруг СМЧД будет моложе своего брата на Земле. Вот тебе по факту путешествие в будущее. Разве нет?
>>133875
Полезность твоих постов зашкаливает. Спасибо.
Аноним 11/11/14 Втр 22:21:30 #90 №133878 
>>133866
>Но проекцией чего является сфера?а?
Проекцией гиперсферы, еблан. Геометрия многомерных пространств существует уже полтора века. И связи с М-теорией она практически не имеет. В общем ясно с тобой все, технарь.
Аноним 11/11/14 Втр 22:31:53 #91 №133882 
>>133877
>Поебать кт там консультант.
Ну конечно обоссаный гуманитарий накормил астрофизика с мировым именем говном на двачах, вот это победа разума над здравым смыслом.
>первокурс ебаный.
Пошли дешевые уловки демагога, не могущего в аргументы.
>Отлично выкрутлся. Мулььивселенные
Вполне себе научная теория, ебанаха. Рассматриваемая учеными, как вполне возможная. Впрочем, другие варианты ты тоже просто проигнорировал. Ты же так просил принести тебе теорию, согласно которой можно в прошлое? Вот, на машина времени Готта, вполне себе теория, но ты не понял ни слова, от того поломался.
>Спасибо. Расходимся.
Пожалуйста, уебывай. Захочешь еще поесть говна, обращайся.
>еометрия многомерных пространств существует уже полтора века. И связи с М-теорией она практически не имеет.
Охуительные истории ИТТ, весь ландшафт теории струн построены на работах Яу, но гуманитарии идут своим путем.
Аноним 11/11/14 Втр 22:34:00 #92 №133883 
>>133413
СТО. Время изменяется при приближении к релятивистским скоростям, насчёт гравитации - не припомню.
Аноним 11/11/14 Втр 22:43:50 #93 №133885 
>>133877
Ебаный ты дурак.
>Ну вот я завтра снмму интерстеллар, н в концовке оине передаст координаты.
Ещё разок.Он всегда передавал координаты.Потом летел.Потом передавал.Летел..Для нас это кажется ракурсией.Потому что время для нас линейно.
>>133878
Я не технарь.И про гиперсферы знаю.
Я про время.
Иди на хуй.
Аноним 11/11/14 Втр 23:16:56 #94 №133888 
>>133439
двачую
помогите разобраться
Аноним 11/11/14 Втр 23:23:59 #95 №133889 
>>133877
>Ну вот я завтра снмму интерстеллар, н в концовке оине передаст координаты. Что мне посоветует Кип Торн для разрешения этой проблемы?

Похуй что ты гуманитарий, это не страшно. Но вот то, что ты долбоёб - это фатально.

ПС мимокрокодил
Аноним 11/11/14 Втр 23:24:34 #96 №133890 
>>133877
>Ну вот я завтра снмму интерстеллар, н в концовке оине передаст координаты. Что мне посоветует Кип Торн для разрешения этой проблемы?

Похуй что ты гуманитарий, это не страшно. Но вот то, что ты долбоёб - это фатально.

ПС мимокрокодил
Аноним 11/11/14 Втр 23:57:31 #97 №133895 
>>133888
>>133439
Вещество не падает на дыру как попало, оно закручивается в диск и выпадает в районе экватора.
Алсо, никто этих дыр никогда не видел, х.з. как они там выглядят на самом деле. Все тории.
В Интерстелларе дырку показали с учетом всевозможных теоретических подробностей, что люто доставило.
Кто может в буржуйский, вот годная статейка.
http://www.wired.com/2014/10/astrophysics-interstellar-black-hole/
Аноним 12/11/14 Срд 01:16:02 #98 №133904 
>>133439
С чего ты взял, что не видим? Мы именно это и видим, падающее вещество, точнее его сияние. Пока не существует инструментов, способных разглядеть собственно горизонт, и мы видим яркое сияние аккреционных дисков, которые более чем на 99% состоят из водорода и гелия, падающего в дыру.
Не забывай, что там действуют мощнейшие во Вселенной силы и чудовищные энергии, пытаться что-то разглядеть на фоне излучения дисков бесполезно. Всё равно что пытаться разглядеть свет карманного фонарика на поверхности Солнца.
Аноним 12/11/14 Срд 05:00:46 #99 №133911 
>>133885
Короче, когда два ученых будут обсуждать, что случится если человек вернётся в прошлое и убьёт себя молодого, второе истошно заорот "этот сосунок не посмеет!", так? Ты тупо линейная лолита
Аноним 12/11/14 Срд 06:17:51 #100 №133915 
>>133400
>Какая должна юыть гравитация на планете? У меня такое ощущение, что просто пиздецовая, т.е. очень большая, при которой человек бы не выжил. Так ли это?

Дело не в гравитации, а в орбитальной скорости планеты: она очень высокая, и если ты путешествуешь с очень высокой скоростью, то твоё время замедляется.

Тут проблема в другом: как они могли погасить скорость и выйти на орбиту близкую к планете. Её скорость лоджна превышать существенную долю световой скорости.
Аноним 12/11/14 Срд 10:20:39 #101 №133926 
>>133450
Так и есть
одна Внутри ядра Галактики
другая Великий Аттрактор
Аноним 12/11/14 Срд 11:18:38 #102 №133935 
14157803188810.jpg
Вы чего, ребята? Вы всерьез обсуждаете этот высер? Где ученые собираются колонизировать планетку у черной дыры потому что на земле вирус, убивающий растения и нечего есть? Где 5-мерные люди из будущего отправляют парня, упавшего в черную дыру за книжный шкаф к его дочке? Дабы он передал ей сакральные знания о природе времени и гравитации которые помогу дочурке сьебать с родной планеты из-за пыльных бурь и колонизировать мир в другой галактике (из-за пыльных бурь и свирепствующих вирусов на земле) - в котором ничего нет? Типа на земле уменьшается количество кислорода в атмосфере, патамушта вирус убивает растения - мы умрем и задахнемся, а давайте терраформируем планетку в другой галактике? - землю то все равно не спасти типа. Какого хуя он болтался у Сатурна в открытом космосе и кто его туда отправил из черной дыры в другой галактике? Пятимерные люди из будущего, которым похуй на время? Просто пиздец - и Нолан еще имеет наглость пиздить о научности этого высера.
Если люди не могут справится с вирусом, убивающим растения - то как они могут переехать в другую галактику и создать новый мир, пригодный для человека с нуля?
Аноним 12/11/14 Срд 11:23:16 #103 №133937 
>>133935
Какой мощный выброс корональной массы у этого красного карлика.
Аноним 12/11/14 Срд 11:34:28 #104 №133939 
>>133935
да
Аноним 12/11/14 Срд 11:42:00 #105 №133943 
>>133939
Но ведь заявления Нолана о научности этого сблева - это плевок в лицо всей науке как таковой. Обыватели сожрут любое распиаренное говно (даже такое где главный герой падая в черную дыру попадает в пятимерное пространство за шкафом в комнате своей дочки, или где для колонизации выбирается планета, вращающаяся очень близко к черной дыре), но есть же предел.
Аноним 12/11/14 Срд 12:02:33 #106 №133958 
>>133943
Еще один дебиловатый.
Аноним 12/11/14 Срд 12:18:15 #107 №133962 
>>133958
>ТЫ ПРОСТА НЕ ПОНЯЛ ВСЕЙ ГЕНИАЛЬНОСТИ ЭТОГО ФИЛЬМА, ВСЕЙ ДРАМЫ
Еще раз, этот фильм - сблев, нет там никакой научности, это даже не научпоп - это бредятина, бредятина и еще раз бредятина. И драмы там нет - там есть убогие сопли, развешанные по всему убогому сюжету про любовь-хуевь и все прочее. Мет Дейман всех наебал чтобы вернутся - вот и все.
И да - в фильме НЕТ науки. Вообще. Там есть стандартное складывание бумаги - типа ВОН ЭДАК МЫ ПРОДЕЛАЕМ ДЫРУ В ПРОСТРАНСТВЕ-ВРЕМЕНИ - это блять я видел в десятке фильмов наверное.
Аноним 12/11/14 Срд 12:24:39 #108 №133970 
Говорят, что корабль должен был сгореть в аккреационном диске чёрной дыры.А разве нельзя было огибать ЧД в другой плоскости, в которой нет этого диска?
Аноним 12/11/14 Срд 12:42:13 #109 №133974 
Нолан:
Зацените поцанчики как прекрасна теоретическая физика!Время относительно.Я вам даже художественное допущение сделаю,ну 1час=7лет,так понятней же.
И давайте представим,что время это нечто такое как пространство.Я вам тут комнату визуализирую.Вы,конечно понимаете,что комната весьма условно отражает теорию многомерности.И это,ну,про мультивселенную я уж так,пару отсылок и все.Грузить не буду.

не быдло:лах,за шкафам в чернай дыре живет.Распидарасит.10ти мерные люди азаза.Кукуруза нинужна.

Лично для тебя >>133911 юродивый ты пидар.
>этот сосунок не посмеет
Именно.Пройди сквозь стену.
Впрочем ты меня заебал,иди на хуй.
Аноним 12/11/14 Срд 12:52:28 #110 №133977 
>>133962
>ВЫСИР!
>ВЫСИР!
>НИНАУЧНА!

Очередной порванный школогумагуманитари не умеющий в аргументы. Фабула фильма почти полностью создана Кипом Торном, но быдло школоло считает, что разбирается в астрофизике гораздо лучше какого-то там Фейнмановского профессора Калтеха и члена научного совета NASA.

>>133970
>А разве нельзя было огибать ЧД в другой плоскости
Купер завалился выше диска, над ним, в фильме это показано. Другое дело, что от этого диска фонит со страшной силой, ну он там и проболтался недолго, да и корабль был спроектирован для длинных путешествий, с учетом высокого уровня радиации.
Аноним 12/11/14 Срд 13:01:14 #111 №133981 
>>133977
>Фабула фильма почти полностью создана Кипом Торном, но быдло школоло считает, что разбирается в астрофизике гораздо лучше какого-то там Фейнмановского профессора Калтеха и члена научного совета NASA.
Твоя аппеляция к авторитетам не ебет тут НИКОГО. Это анонимная имиджборда. Если член научного совета NASA создаёт откровенно бредовую фабулу, это не значит, что фабула на самом деле блестящая, а сосачевские дебилы её не замечают.
Аноним 12/11/14 Срд 13:04:52 #112 №133983 
>>133981
>а сосачевские дебилы её не замечают.
Сосачевские дебилы не могут в аргументы. Так что проблема в дебилах с сосача, а не в апелляции к авторитетам.
Аноним 12/11/14 Срд 13:06:13 #113 №133984 
>>133935
>Вы всерьез обсуждаете этот высер?
Фильм не претендует на строгую научность, в любом случае (это же массовое кино).
>Пятимерные люди из будущего, которым похуй на время?
Ты не можешь с уверенностью заявлять, что это невозможно, поэтому не позорься вскукареками, пожалуйста.
>Если люди не могут справится с вирусом, убивающим растения - то как они могут переехать в другую галактику и создать новый мир, пригодный для человека с нуля?
А вот тут я с тобой соглашусь.
Аноним 12/11/14 Срд 13:08:36 #114 №133985 
>>133983
Ты не понял. Может, Нолан вообще консультациями практически не пользовался, может, специально переделал всё под собственное видение сценария. А кукарекать о том, что фильм делали СПЕЦИАЛЬНО ОБУЧЕННЫЕ ЛЮДИ, и там не может быть ляпов - неправильно.
Что, впрочем, не отменяет того, что ты и школьник, с которым ты спорил, только и умеете поливать друг друга говном.
Аноним 12/11/14 Срд 13:10:09 #115 №133986 
>>133974
Теоретическая физика? Но позвольте - единственный момент где в фильме проскальзывает теоретическая физика - момент со складыванием бумажки. Все. Больше там физики НЕТ. А есть там как ты выразился "допущения" - которые там на каждом шагу, всюду и везде.
- Ой, давате-ка сядим на плонету, которая очень близко к ЧД - может там новый дом для человечества?
- Ой, я ни умер, тут пятимерные люди меня заслали в комнатку моей дочки, ой, я ебу в рот пространство-время, ЧД? - ХА ХА ХА.
- Ой, лежу в больничке - болтался вокруг Сатурна в космосе в одном скафандре, и меня спасли. Как я там оказался - С ПОМОЩЬЮ ОСОБОЙ МАГИИ ПЯТИМЕРНЫХ ЛЮДЕЙ
>>133977
Фабула фильма? Что блять? Там нет астрофизики - ее не видно за соплями и бредом.

Аноним 12/11/14 Срд 13:55:13 #116 №134000 
>>133986
>Что блять? Там нет астрофизики
Порванные школьники без аргументов меня начинают веселить.
>ее не видно за соплями и бредом.
Сейчас, если попрошу назвать хороший научно-фантастический фильм на космическую тематику то вскрою целый зоопарк говноедов.


Аноним 12/11/14 Срд 14:08:50 #117 №134006 
>>134000
Повсюду школьники, псевдо-научный сблев Нолана - шедевр, у одного тебя есть аргументы. Фильм создавался при участии Кипа Торна, а все хейтеры - школьники/гуманитарии/имбецилы и у них нет аргументов, в отличие от тебя. Успокоился?
Аноним 12/11/14 Срд 14:13:26 #118 №134011 
Сценарий писал точно не физик - потому что когда группа ученых садится на планетоид, который кружится в вальсе смерти вокруг черной дыры - причем очень близко, чтобы узнать - можно ли создать на нем новый дом для человечества - это довольно странно.
Аноним 12/11/14 Срд 14:32:27 #119 №134021 
>>133986
Ты до сценария доебался.
И поступков персонажей.
Критики теорий в твоем посте - нет.

Это:
там физики НЕТ
@
Там нет астрофизики
Не конает.
Аноним 12/11/14 Срд 14:41:28 #120 №134025 
>>133915
Кажется, про влияние скорости на время гласит СТО.
А зависимость времени от искривления пространства, от 'гравитации' изучает ОТО.
И оба этих фактора влияют одновременно. Где-то приводились даже расчёты для спутника, вроде из-за скорости время идёт медленнее, чем на поверхности Земли, а из-за
меньшего искривления пространства на орбите - быстрее. И в итоге эти "дефекты" оставляют разницу в десятки микросекунд.
Аноним 12/11/14 Срд 14:54:27 #121 №134036 
>>133943
Еще оди адекват в треде! Все под горизонт! ОП
Аноним 12/11/14 Срд 14:57:02 #122 №134041 
>>134006
Нет, у тебя нет аргументов, почему фильм сблев, и логика у тебя отсутствует. Либо ты специально передергиваешь, либо ты просто имбецил со справкой. От тебя только петушиный крик стоит, что сблев, сблев, сблев. Порванка, зашивайся.
>>134011
Физики не пишут сценариев, инавче сценарии были бы говном. Физиков приглашают за консультациямми на предмет не противоречия визуализации и событий законам физики.
Они поперлись туда на основе данных, которые отослал автоматический зонд, есть атмосфера и вода, о том, что там воны с километр бегают, могли бы догадаться, конечно. Тем не менее, планета не на столько близка была к ЧД, что бы ее распидорило. Какая разница, вокруг чего вращаться, мы же падаем на Солнце уже 4.5 млрд лет. И ничего.
>>133915
>Её скорость лоджна превышать существенную долю световой скорости.
К сожалению к счастью блеадь на самом деле не известны ни размеры, ни расстояния, а по угловым тоже не определишь, так что ни размера ЧД, ни орбиты, ни массы там не указано, как высчитывать все эффекты и скорости, без данных от слова вообще, мне не понятно.
Аноним 12/11/14 Срд 15:29:56 #123 №134069 
14157953967220.jpg
Аноним 12/11/14 Срд 15:31:39 #124 №134071 
>>134041
Такое ощущение что тебя проплатил Нолан - судя по масштабной рекламной кампании этого говна - я не удивлюсь что это так и есть.
>Физиков приглашают за консультациямми
Почему если они консультировались у физиков/астрофизиков фильм получился настолько бредовым?
>не на столько близка была к ЧД
>Какая разница, вокруг чего вращаться
Да ты что? И вообще, жить на такой планете - хорошая идея, да? Я же говорю - ты как зомби. просто обыватели твердят в один голос - ШЕДЕВР, и ты - миллионы идиотов ведь не могут ошибаться.
Аноним 12/11/14 Срд 15:33:32 #125 №134073 
>>134041
>Физиков приглашают за консультациямми на предмет не противоречия визуализации и событий законам физики
Как будто Нолан настолько ебанутый, чтоб еще и физикам платить за консультацию. Достаточно подобный тред поскроллить, чтоб обеспечить антинаучной хуетой любой сценарий
Аноним 12/11/14 Срд 15:36:04 #126 №134076 
А почему в фильме не рассказано что за теорию всего/тысячное имя бога/великую тайну о том что было до большого взрыва открыл робот в черной дыре? Вот там вверху пиздят о некой теории в фильме? ГДЕ ОНА? Про то что время течет тем медленнее, чем глубже гравитационный колодец - великая теория, на которой основан этот глупый фильм?
Аноним 12/11/14 Срд 15:54:32 #127 №134088 
Я как-то падал с пацанами в черную дыру, мы ощутились в галактике М3, там с местными пацанами зазнакомились, шашлыки, водочка, телочки, витя ака. Каждый год теперь падаем нет-нет, тачка была лада гранта, расточенная. Никаких косяков вообще, ходовка по децылу терпела, но все нармулики, паца.
Аноним 12/11/14 Срд 16:10:10 #128 №134101 
>>134076
>А почему в фильме не рассказано что за теорию всего
Пытается же.Нолан,какбы,указывает пальцем и говорит: Смотри как все может быть.Не хуевые расклады,а?
А ты смотришь на палец и ,такой:хули ты мне палец показываешь,лалка.Палец как палец.Нахуй ты в шкаф залез?
Аноним 12/11/14 Срд 16:14:19 #129 №134106 
Алсо, есть техспецификация шкафа? Может шкаф из антииматерии?
Аноним 12/11/14 Срд 16:54:19 #130 №134158 
>>134071
>настолько бредовым?
По прежнему жду аргументов не уровня "говно потому что говно"
>И вообще, жить на такой планете - хорошая идея, да?
Да, заебись, че. Если бы не приливы с километр, с другой стороны - серфинг там охуенный, наверно. Курорт можно сделать.
>Я же говорю - ты как зомби. просто обыватели твердят в один голос - ШЕДЕВР
Я же говорю - ты как зомби. просто говноеды твердят в один голос - ГОВНО.
>>134076
>Как будто Нолан настолько ебанутый, чтоб еще и физикам платить за консультацию.
О, залетная ебанаха, которая даже не потрудилась прочитать тред.
http://www.youtube.com/watch?v=jQye2XkvDpo
>>133985
Еще один дебил, не читавший тред, сколько же вас , уебков, слетелось.
>что фильм делали СПЕЦИАЛЬНО ОБУЧЕННЫЕ ЛЮДИ,
Фильм делали специально обученые люди по изготовлению художественных фильмов.
>и там не может быть ляпов - неправильно.
Никто не говорит, что там нет ляпов, я тебе открою страшную буржуинскую тайну - художественное кино - это всегда условности и допущения, иначе смотреть это никто не будет. Хотите строгой научности - смотрите документалки, нахуя школоло суется свинным рылом в калашный ряд - мне не понятно.
>>134076
>А почему в фильме не рассказано что за теорию всего
Как раз про теорию всего там талдычат весь фильм. Отсутствие решения уравнения квантовой гравитации является как раз целью поиска, что бы спасти всех. Герой ее находит в ебучем шкафу. Ну шкаф и шкаф. В нем, кстати, жирные намеки на теорию суперструн, и теорию Мультивселенной, но вы видите там только шкаф ебучий, вы вообще жопой фильм смотрите и в художественные образы не можете совсем?
>о том что было до большого взрыва
Нет задач
>Вот там вверху пиздят о некой теории в фильме? ГДЕ ОНА?
Прикинь, нет ее. 60 лет бьются и безрезультатно. М-Теория, суперструны, вот это все. А по факту нихуя не выходит.
>на которой основан этот глупый фильм?
Блядь, еще один ищет говно на шоколадной фабрике. Фильм вообще не об этом.
>>134101
Вот этот все правильно понял.
>>134106
Спецификация шкафа такова, что шкафа вообще может не существовать. Возможно, что это визуализация образов, что бы Купер с ума не сошел и ебался со своим посланием в привычной для него обстановке, дергая за струны.
>>134106
>Может шкаф из антииматерии?
Не, тогда бы Купера распидорило, вместе со шкафом.
Аноним 12/11/14 Срд 17:19:02 #131 №134184 
>>134101
Может он и сам не знал что-бы такое придумать чтобы зацепило обывателей, поэтому не поставил точку в этом говно-фильме.
Аноним 12/11/14 Срд 17:22:07 #132 №134188 
>>134158
Лол, я (в отличие от тебя) не любитель навешивать ярлыки - но по-моему восторженный школьник как раз ты. Зачем ты агришся на всех кому смешон твой восторг этим фильмом? Зачем эти простыни текста?
Этот фильм рассчитан на идиотов, на стадо потребителей, которым нужна красивая картинка и незамысловатая история.
Аноним 12/11/14 Срд 17:29:40 #133 №134196 
>>134158
>Вы дебилы, ничего ни понили
>Еще адин дебил
О господи, тебе 14?
>Фильм вообще не об этом.
Хорошо. О чем фильм? О Глупости тех кто писал дебильный сценарий с черной дырой и планетами вокруг нее, которым суждено стать новым домом для лысых приматов?
>>134158
>Спецификация шкафа такова, что шкафа вообще может не существовать. Возможно
>Возможно
Давай додумывать, ну.
>это всегда условности и допущения, иначе смотреть это никто не будет.
Так тебе про это и говорят - там одни условности и ляпы - зачем ты сраться то начал со всеми?
Аноним 12/11/14 Срд 17:39:07 #134 №134215 
>>134196
Я не >>134158 но.
>там одни условности и ляпы
>условности
логические допущения.
>ляпы
Например?
Что там не укладывается в реалии?
Аноним 12/11/14 Срд 18:11:39 #135 №134272 
>>134196
Никто, ни один пизденыш так и не сказал, что конкретно в этом фильме не научно. Вы все уперлись в шкаф и все.
>О господи, тебе 14?
Это вам, ебанахи, по 14, вы тред не читали, в аргументы не можете. Когда вы рветесь без аргументов вы скатываетесь до дешевых приемчиков "сколько тебе лет", ты уже третий такой дебил, или четвертый.
>О Глупости тех кто писал дебильный сценарий с черной дырой и планетами вокруг нее, которым суждено стать новым домом для лысых приматов?
Вот, завел снова свою мантру. Вот я тебя спрашиваю, о чем фильм? Там, кстати несколько раз говорится чуть ли не прямым текстом об этом.
>Давай додумывать, ну.
Еще один не может в образы и не понимает, чем документалистика отличается от художественного фильма.
>там одни условности и ляпы
Еще раз, ебанаха, какие именно там ляпы и условности, которые не научны? Что конкретно там не научно, кроме ебучего шкафа, который вам взорвал жопу?

Аноним 12/11/14 Срд 18:17:18 #136 №134275 
>>134272
Не истери.
И да, судя по всему тебе 14, восторженный ебанат.
Аноним 12/11/14 Срд 18:20:45 #137 №134282 
>>134275
Аргументов не будет, еще один обоссан, замечательно. Хороший сегодня день, куча порванных гуманитариев.
Аноним 12/11/14 Срд 18:27:57 #138 №134292 
>>134282
Молодец, показал проклятым хейтерам где их место. Как они смели обсирать лучший фильм, который ты видел в своей жизни? Мамка будет тобой гордиться.
Аноним 12/11/14 Срд 18:44:07 #139 №134327 
>>134196
Где ляпы?
>>134272
А ты,в натуре, не истери!
Вот за это:
>Что конкретно там не научно, кроме ебучего шкафа
Пиздить надо.
Это как сказать,что решение задачи про поезд,следующий из А в Б не научно потому,что училка ёбнутая.Хули тетрадку с каракулями показывает?При чем тут поезд?Там бумага.Хуета же.Поезд в тетрадке,лол.Прикинь:у неё поезд на листке бумаги проехал.Пиздец она ёбнутая.
Аноним 12/11/14 Срд 20:23:19 #140 №134571 
>>134041
Хорошо, ты тут пишешь про отсутствие аргументов, я попробую провести нить. Выше в треде выяснилось, что теоретическая планета может вращаться исключительно вокруг СМЧД. Иначе разрыв приливными силами гарантирован. Тут всплывает такой момент. Ближайшая СМЧД от Земли находится в центре галактики. 26000 световых лет? Ты так настаиваешь на научности, что я предполагаю отсутствие FTL-движения. То есть 26000+ лет они летели до точки назначения и не постарели? То есть со скоростью около 99,9999999999%+с? Ну так вот, теоретического обоснования сейчас для разгона какого-либо КА до такой скорости нет. Это уже ненаучно. Ты скажешь, что это не про центральную галактическую СМЧД, а про какую-нибудь поближе. Ну так вот, необходимый параметр массы для дыры, вокруг которой может вращаться планета ~1000 солнечных. Такой объект просто невозможно пропустить в наблюдаемом секторе галактики даже на данном этапе наблюдений. Существование оного уже ненаучно. Далее сама планета. Очевидно, что существование ее возможно исключительно в формате происхождения гравитационным захватом. И чтобы не быть поглощенной ЧД и выйти на орбиту ей необходимо иметь также околосветовую скорость относительно ЧД. Ладно, это еще допустимо в рамках современных теорий. Но тысячекратное замедление времени возможно только на предельно низкой орбите, практически касающейся горизонта событий. Сам же писал, что для всех ЧД характерен аккреационный диск. Что станет с планетой после путешествия по такому пиздецу? Ок, под углом, говоришь Миклуха подлетал? Но планета не может вращаться где-то выше по широте, она обязательно будет проходить через экваториальную плоскость, ибо стабильное вращение вне плоскости центра масс невозможно. Что с ней сделает циркулярка высокоэнергетических частиц наверняка сказать нельзя, но что можно сказать наверняка, так это то, что волн и атмосферы там не останется. Ок, допустим ЧД скушала все вещество поблизости и никаких частиц вокруг нее не летает. Но мимопроходящий камешек, который наверняка хоть раз за астрономически значимый период стукнет планету на встречной скорости около с, не оставит от нее никакого подобия эллипсоида. Хорошо, пусть даже камешки не летают. Тогда возьмем тепловой баланс. Почему на планете жидкая вода? Ведь от жадной ЧД никаких излучений не дождешься. Существование планеты, внутренней тектонической энергии которой достаточно для поддержания жидкого океана на поверхности также не научно, слишком много этой самой энергии для этого нужно. Итого крест на "научности" кинца ставит эта планета. Если бы не она, то могла бы получится такая себе псевдо-философская притча с намеком на достоверность. Ведь хуй знает, что там под горизонтом. Пятимерных алиенов только не надо пихать во все поянения, ок? А то любую хуету можно объяснить их вмешательством. Можно назвать сие поделие научной фантастикой, только научной это ее не сделает. Science-fiction куда более точный в данном случае термин.
Только давай без Кипов Торнов и клейма гуманитария, по пунктам поясняешь за теоретические обоснования вышеупомянутых допущений.


Аноним 12/11/14 Срд 20:27:36 #141 №134575 
14158132570000.jpg
Аноним 12/11/14 Срд 20:43:16 #142 №134584 
14158141968290.jpg
Впервые мой тред рождает такие дискуссии
Аноним 12/11/14 Срд 20:43:24 #143 №134585 
>>134571
Ебать.
Аноним 12/11/14 Срд 20:45:03 #144 №134588 
>>134088
С пятимерными пацанами?
Аноним 12/11/14 Срд 20:47:08 #145 №134592 
>>133400
Пацаны, а у кого-то встал от оппика? У меня че-то лежит, хотя мозг грит, что любопытно.
мимо нонфапер
Аноним 12/11/14 Срд 20:50:17 #146 №134596 
>>134592
Это моя бывшая. Моар как-то не по-джентельменски кидать
Аноним 12/11/14 Срд 20:52:24 #147 №134597 
>>134571
Бро, ты не смотрел фильм просто. Они не летели - им пятимерные посоны из будущего сделали червоточину (так в фильме - это не шутки блять).
Этот фильм - жуткий бред, не советую тратить деньги, кстати, если надумаешь пойти.
Аноним 12/11/14 Срд 21:02:27 #148 №134605 
она может прецессировать, как орбита меркурия у Солнца, например.
Тоже заинтересовался вопросом, есть подозрение, что нет. В первом ландавшице, кажется, исследовался вопрос, при каком степенном законе силы тяжести есть стабильные замкнутые орбиты, и показывалось, что только при r^-2
Аноним 12/11/14 Срд 21:05:59 #149 №134606 
про чо тред?
Аноним 12/11/14 Срд 21:07:42 #150 №134608 
14158156620540.jpg
>>134606
Про всю хуйню.
В сосновном ответы на вопросы по фильму интерстеллар, немного про сиськи, немного про гуманитариев
Аноним 12/11/14 Срд 21:09:59 #151 №134610 
>>134606
Про ровных посонов с пятого измерения, всегда приходящих на помощь человечеству же.
Аноним 12/11/14 Срд 21:18:01 #152 №134619 
14158162816930.gif
>>133439
Гравитация и энергия изменяют кривизну пространства-времени. Свет распространяется по наиболее короткой траектории (свет=информация=влияние физических объектов друг на друга). При маленьких энергиях и массах самая короткая линия между двумя точками - прямая. Когда масса или энергия достаточно велики, то для очень удалённого наблюдателя будет казаться, что свет движется по кривой траектории, а не напрямик. Отсюда, например, наблюдающееся экспериментально гравитационное линзирование.

Чёрная дыра искривляет пространство настолько сильно, что появляется условная граница - качественно как на рисунке (график функции 1/x^2).

Расстояние для света надо мерить по синей линии, а кажущееся нам (очень далёкому наблюдателю) расстояние - по оси X. Допустим, слева внутренность чёрной дыры, справа - внешний мир.
Если ты захочешь взять и померить расстояние между двумя точками слева, ты должен найти длину отрезка синей линии между ними.

Но если ты захочешь померить расстояние между одной точкой слева и другой - справа, то ты получишь бесконечно большое расстояние (график-то уходит в x=0 в бесконечность с обеих сторон). В результате, свет может ходить только по области слева, только по области справа совершенно свободно, но никогда не сможет пересечь саму границу раздела, т.к. для него она бесконечно далека.

Отсюда и берётся этот горизонт.
Но самое интересное, что несмотря на дикое искривление пространства, сам по себе свет этого не заметит, для него вся картина вывернута наизнанку. Он путешествует в плоском пространстве по своей прямой линии, но горизонт событий для него находится просто на бесконечно большом расстоянии.
Отсюда, кстати, следует и то, что достаточно маленького наблюдателя силы гравитации не разорвут на части.
Аноним 12/11/14 Срд 21:41:46 #153 №134647 
>>134596
Я и не прошу моар. Просто интересно стало на мнение других. Хуец так и не дернулся на нее. Такие дела.
Аноним 12/11/14 Срд 21:53:43 #154 №134658 
>>134647
дык тут эффекты ОТО обсуждаются, на них стоит во сто крат сильнее
Аноним 12/11/14 Срд 22:10:15 #155 №134667 
14158194156990.jpg
>>134647
Ну и правильно
Аноним 12/11/14 Срд 23:00:14 #156 №134685 
14158224141260.jpg
Джорджу Лукасу сказали что в космосе корабли не могут взрываться со звуком на что он ответил "а у меня взрываются, азазаза"

А вы говорите фильм ненаучный, планеты не могут так близко находиться к черной дыре которых на самом деле нет
Аноним 12/11/14 Срд 23:07:35 #157 №134687 
14158228554590.png
>>134685
>которых на самом деле нет
Not this shit again
Аноним 13/11/14 Чтв 00:03:53 #158 №134705 
>>133911
Не "не посмеет". Скорее не сможет.
Аноним 13/11/14 Чтв 00:38:52 #159 №134717 
Ты опять напился,ебанашка?
Аноним 13/11/14 Чтв 00:39:39 #160 №134718 
>>133911
Есть мнение ряда ученых, среди них наш любимый колясочник, что изменить прошлое нельзя.
Аноним 13/11/14 Чтв 00:40:30 #161 №134720 
>>134215
>Например?
>Что там не укладывается в реалии?
Сука, какой же ты ебанат.

Там 90% фильма в реалии не укладывается. И все эти 90% тебе никто не станет на двачах расписывать на самом деле уже дохуя расписали, но в силу своей ущербности ты не видишь, что предоставленные аргументы хоть сколько-нибудь существенны для того, чтобы стать препятствием для такого полёта. Если нужно больше АРГУМЕНТОВ!!!111 и аргументов к аргументам о том, почему эти аргументы аргументны, то кури учебник по астрономии/астрофизике, есть куча статей/учебников, в которых понятным даже имбицилу языком описывается всё, начиная от ЧД и кончая ОТО, СТО и так далее.

Ну а вообще-то с художественной точки зрения фильм очень неплох. Хорошая картинка, отличная озвучка, эмоциональная составляющая также доставляет. То, что они там кукарекали про небывалую научность фильма - это да, пиздёж. Вообще главная дилемма, стоявшая перед создателями фильма - это найти баланс между двумя стульями: реалистичностью и количеством различных пренебрежений физическими законами/явлениями читай "между строгой документалкой, где всё будет по науке, и откровенным бредом, где на научную составляющую вообще можно положить болт, как например, в "Стражах Галактики".
Аноним 13/11/14 Чтв 01:07:24 #162 №134726 
что с тобой?я не враг,бро,успакойся.
>Там 90% фильма в реалии не укладывается.
Конкретно-что?

Аноним 13/11/14 Чтв 01:26:18 #163 №134728 
>>134571
>Только давай без Кипов Торнов и клейма гуманитария
Окей, наконец-то кто-то снизошел до аргументированного спора. Отвечаю без метания говна:
Фильм ты не смотрел, это понятно.
>что теоретическая планета может вращаться исключительно вокруг СМЧД. Иначе разрыв приливными силами гарантирован.
Не гарантирован, планета может обращаться вокруг обычной ЧД, так же как Земля обращается вокруг Солнца. Если ЧД вращается, то возможна стабильная орбита.
С падением в ЧД действительно есть проблема, если ЧД окажется слишком маленькой, то чувака распидорасит, однако, посмотрев кино даже на экране видно, что ЧД хоть и не сверхмассивная, но далеко не компактного размера. Теоретически он мог упасть в дыру, не будучи разорванным к хуям. Это не противоречит физике.
>То есть 26000+ лет они летели до точки назначения и не постарели? То есть со скоростью около 99,9999999999%+с? Ну так вот, теоретического обоснования сейчас для разгона какого-либо КА до такой скорости нет. Это уже ненаучно. Ты скажешь, что это не про центральную галактическую СМЧД, а про какую-нибудь поближе.
Они летели через стабильную кротовую нору, кротовые норы не противоречат физике. Проблема в стабильности кротовых нор, однако, если цивилизация смогла производить, или перенаправлять достаточное количество энергии, дохуя на самом деле то кротовую нору можно сделать стабильной. Проблема не в теории, а в технологиях.
>Ну так вот, необходимый параметр массы для дыры, вокруг которой может вращаться планета ~1000 солнечных.
Хотелось бы ссылку на исследование, в котором говорится об этом. Планета может вращаться вокруг и обычной ЧД, в 4 массы Солнца, например. Откуда эта 1000?
>Такой объект просто невозможно пропустить в наблюдаемом секторе галактики даже на данном этапе наблюдений.
Еще один факт, который говорит о том, что фильм ты не смотрел, червоточина открывалась в другой галактике. Алсо, посмотри, какое количество объектов является кандидатами в черные дыры на текущий день, их считанные единицы, если не брать в расчет СМЧД в ядрах других галактик.
>Далее сама планета. Очевидно, что существование ее возможно исключительно в формате происхождения гравитационным захватом.
Тут соглашусь, маловероятно, что планета может выжить при взрыве сверхновой, учитывая, что там недалеко еще НЗ. Получается, что коллапсара было два.
>ЧД и выйти на орбиту ей необходимо иметь также околосветовую скорость относительно ЧД/
Если ЧД таки не сверхмассивная, то угловая скорость будет не околосветовой, но да, в любом случае довольно высокой. Для справедливости отмечу, что при захвате планеты у нее не будет стабильной орбиты, по крайней мере не будет круговой точно, так что тут действительно косяк.
>Сам же писал, что для всех ЧД характерен аккреационный диск
Нет, я этого не писал, но диск в фильме присутствует. Аккреционный диск бывает только тогда, когда ЧД есть что похавать.
В фильме он есть и показан, кстати с учетом всех тонкостей понимания физических теорий.
>Но планета не может вращаться где-то выше по широте, она обязательно будет проходить через экваториальную плоскость
Купер не планета, тем более радиус орбиты планеты может вполне быть больше радиуса диска, как в фильме и показано, кстати.
>Ведь от жадной ЧД никаких излучений не дождешься.
Как я понял из фильма, излучение идет от диска, и довольно сильное
>внутренней тектонической энергии которой достаточно для поддержания жидкого океана на поверхности также не научно
Откуда по твоему там волны километровые? Это приливные волны, планета разогревается приливными силами. Тут,как раз все четко и никаких противоречий. У нас в примере Энцелад и Европа, которые имеют подледные океаны с жидкой водой, именно из-за приливных сил.
>Ведь хуй знает, что там под горизонтом.
Вот именно, что хуй знает, поэтому я вообще не понимаю, как можно что-то опровергать заведомо неопровержимое, так же как и утверждать то, что заведомо не возможно проверить. Зато это развязало руки режиссеру, который въебал туда шкаф. Норм отмаза, я считаю. Ну шкаф и шкаф, хуй с ним. Кому-то печет, я считаю, что режиссер имеет право на образы и метафоры.
>Science-fiction куда более точный в данном случае термин
>Можно назвать сие поделие научной фантастикой, только научной это ее не сделает.
Ох, лол, что же ты делаешь, анон!? Science-fiction - это и есть научная фантастика!

Резюмирую:
Из всего, что было в фильме не совсем научного и малоправдоподобного - это стабильная орбита планеты вокруг ЧД, да еще и с вторым компонентом в виде НЗ. Это действительно сделано в ущерб физике, в пользу сюжета.
Второе, менее интересное - это возможность выживания при падении в ЧД. Не известны параметры ЧД и точно сказать распидорит, или нет - не возможно.
Третье - это излучение от ЧД, точнее от диска, которое скорее всего зажарило бы Купера и стерилизовало бы планеты, ибо жесткий рентген такой жесткий.
Все, не так уж и много ляпов получилось.



Аноним 13/11/14 Чтв 01:27:38 #164 №134729 
>>134726
>Конкретно-что?
Да он сам не знает, еще один ебанат, не могущий в аргументы.
Аноним 13/11/14 Чтв 01:56:30 #165 №134735 
Почему еще никто не кинул это ссылку?
http://ikjyotsinghkohli24.wordpress.com/2014/11/07/on-the-science-of-interstellar/
Аноним 13/11/14 Чтв 02:36:56 #166 №134736 
14158354166980.png
>>134728
>планета может обращаться вокруг обычной ЧД, так же как Земля обращается вокруг Солнца
Не на том расстоянии, что я видел в трейлере.
>кротовые норы не противоречат физике
Не смеши, все, что нарушает принцип причинности на данный момент не научно.
>Откуда эта 1000
Приблизительный параметр, при котором горизонт событий "обгоняет" предел Роша для твердого планетарного объекта. На самом деле меньше, просто считать впадлу было, много матана.
>В фильме он есть и показан, кстати с учетом всех тонкостей понимания физических теорий.
Ну тогда очевидно, что это шрёдер никакая планета не переживет. Воды с атмосферой точно не будет.
>тем более радиус орбиты планеты может вполне быть больше радиуса диска, как в фильме и показано, кстати
Так на пике не та планета, о которой мы говорим? Ну да похуй. Многомиллионное замедление времени возможно только на околосветовых скоростях и предельной близости к горизонту событий, так что вариант с не сверхмассивной дырой однозначно отпадает.
>так же как и утверждать то, что заведомо не возможно проверить.
В этом и суть. Стоило Нолану ступить на путь собственного виденья, как фильм сразу перестал быть научным. Это в оп-посте и имел ввиду гуманитарий. Что даже на его недалекий взгляд очевидно, что фильм - одно большое допущение, опирающееся лишь на удобные физические гипотезы, а порой, для художественного выражения и вовсе игнорирующее законы. Зачем ты кидаться на всех начал? Нельзя называть научным фильм, который предлагает лишь виденье некоторых людей, основанное на творческих порывах. Творчество - это прекрасно, но оно практически не пересекается с научным подходом.
>Science-fiction - это и есть научная фантастика!
Вот зачем ты всех за идиотов принимаешь? Все, что я хотел сказать, так это то, что английский вариант имеет менее расплывчатое лексическое значение.





Аноним 13/11/14 Чтв 03:03:47 #167 №134739 
>>134729
Перечитай тред, уебан, тут дохуя конкретики описали.
Аноним 13/11/14 Чтв 03:13:02 #168 №134741 
>>134726
На хотя бы вот это http://science.d3.ru/comments/618794/
Аноним 13/11/14 Чтв 03:22:22 #169 №134742 

>>134739
Какой конкретики, уебок? Одни "азаза шкаф бля!"
Все, только один анон отписался конкретно, что ему не понравилось, остальные только визжали, что говно и все тут.
>Не на том расстоянии, что я видел в трейлере
Не смотрел, но осуждаю. Пиздец, о чем вообще с тобой разговаривать, если ты вообще сабжа в глаза не видел?
>на данный момент не научно.
Не может быть научно в один момент и ненаучно в другой, по факту кротовые норы ОТО не противоречат, так что не зачет.
>Приблизительный параметр
>просто считать впадлу было
То есть только из твоей головы, с потолка.
>Многомиллионное
Какое-какое?
>Стоило Нолану ступить на путь собственного виденья
Любой, абсолютно любой фильм - это собственное видение режиссера, на то это и художественное кино, иначе - документалистика.
>Это в оп-посте и имел ввиду гуманитарий
Ты теперь не только за себя, но и еще за другого парня?
>Нельзя называть научным фильм, который предлагает лишь виденье некоторых людей, основанное на творческих порывах
Любой художественный фильм - основывается на творческих порывах, назови хоть один научно-фантастический фильм, где все охуенно гладко с точки зрения логики, здравого смысла и физики?
>Все, что я хотел сказать, так это то, что английский вариант имеет менее расплывчатое лексическое значение.
Нет, дружочек, это прямой перевод одного понятия, с одного языка на другой. Не верти жопой.
>>134741
Там точно такие рассуждения, уровня спейсача.
Аноним 13/11/14 Чтв 03:41:25 #170 №134743 
>>134742
>Там точно такие рассуждения, уровня спейсача.
А тебе, блять, что нужно, вывод формул? Экспериментальные опровержения? Ты заебал уже.
Аноним 13/11/14 Чтв 04:07:19 #171 №134745 
14158408397670.jpg
14158408397811.jpg
14158408397862.jpg
14158408397873.png
>>134743
>Если это свет от аккреционного диска, то он должен быть как минимум в рентгеновском диапазоне.

Ну это разве не пиздец? Как будто аккреционный диск излучает исключительно в рентгене.
Достаточно?

Алсо, там доставляющий срач в камментах.
Аноним 13/11/14 Чтв 04:19:42 #172 №134747 
14158415823110.jpg
А разгадка проста. Наука в фантастике всегда являлась и будет являться оболочкой для человеческой истории. Сайфай любой степени твердости без человекопроблем - это документалка или фильм для аутистов, которые даже не припоинить. Яркий пример человекоориентированной сайфай высокой твердости - космическая одиссея. Но если начать разбирать ее с точки зрения современной науки, логики, и плотдевайсов то тут поднимется утиный вой.
Аноним 13/11/14 Чтв 04:32:24 #173 №134749 
>>134747
Вулканец пиздеть не станет.
Аноним 13/11/14 Чтв 06:24:29 #174 №134756 
>>134584
да ты просто открыл филиал мувача.
Аноним 13/11/14 Чтв 07:45:30 #175 №134759 
14158539309500.jpg
>>134736
бро, ты по скриншоту умеешь определять расстояния? ну-ка определи по картинке расстояние от планеты до звезды. Ответ дай в единицах звёздных диаметров.

> Не смеши, все, что нарушает принцип причинности на данный момент не научно.

окей, назови хоть одну хорошо экспериментально проверенную область физики, где мат.аппарат или аксиоматика требует течение процессов только в одном направлении во времени, и не позволяет замену t на -t. Кроме термодинамики.
Далеко не всё так очевидно. Существования замкнутых мировых линий есть следствие текущего понимания ОТО. Прямых запретов на них из других теорий не следует. Пока нет экспериментальных подтверждений их существования, или существования явлений, прямо их запрещающих, мы не можем этого утверждать. Азазаза, "квантовые эффекты должны уничтожать такие кротовые норы" - может быть, но квантовой гравитации, проверенной экспериментально тоже пока нет.
Принцип причинности сохранить всем хочется. Как когда-то хотели сохранить эфир или относительность Галлилея.

> Ну тогда очевидно, что это шрёдер никакая планета не переживет. Воды с атмосферой точно не будет.
Никто и не говорит про планету внутри аккреционного диска. Есть обязательные требования к размеру диска? Почему он не может существовать в зоне, где, допустим, лоренц-фактор 10^11 ? ЧД, возникшая в ранней вселенной, аккреционный диск из вещества мимо пролетавшей звезды, попавшей в неё туда же, к 15-ти миллиардному году от Божественного творения уже может быть довольно близким к горизонту событий.

> Многомиллионное замедление времени возможно только на околосветовых скоростях
Либо при больших ускорениях == силе тяжести. Но да, около горизонта событий.

И да, в фильме её называли сверхмассивной.


И вообще, фильм, может и говно, но что бы ещё заставило сотни гуманитариев вообще задумываться о пространстве, времени, ОТО, и современных проблемах физики? Ящетаю, вин уже хотя бы поэтому.
Не самое дибильное, не самое ляповое кино, использующее в сюжете невероятно сложные вещи, познанные человечеством на листке бумаги, и которые, весьма вероятно, никто никогда не сможет потрогать вживую.
Аноним 13/11/14 Чтв 12:02:24 #176 №134821 
Бредовость фильма даже не в убогой идее с пятимерными людьми (почему пятимерными и почему людьми - я то всегда надеялся что роботы с исскуственным интеллектом - это следующий шаг, который был предначертан еще во времена большого взрыва когда все формировалось) а в том что герои с самого начала занимаются какой то хуитой - летят колонизировать планеты в другой галактике из-за каких то проблем на земле, ведь им не угрожает не знаю взрыв сверхновой или удар какого нибудь здоровенного астероида - им угрожает какой то вирус и опустынивание. Летят они почему то на планеты. которые вращаются вокруг ЧД - это ли не пиздец?
Кстати жизнь на планетах вокруг ЧД - невозможна, если ЧД расположена в центре галактики - то невозможна вдвойне.
Аноним 13/11/14 Чтв 12:29:42 #177 №134830 
>>134742
>ОТО не противоречат
Не единой ОТО живет современная физика.
>То есть только из твоей головы
Да.
>с потолка
Нет. Метод я написал.
>Ты теперь не только за себя
Я изначально против твоей агрессии. Тут не для ноланофанатизма доска.
>абсолютно любой фильм - это собственное видение режиссера
Я и не спорил, только вот когда это виденье переход границы нынешней теоретической физики, то научным такой фильм уже ни с какой стороны не является.
Да даже какие-нибудь "Эллизиум" или "Пандорум" более научны, чем "Интерстеллар".
>назови хоть один научно-фантастический фильм, где все охуенно гладко с точки зрения логики, здравого смысла и физики
А зачем вообще научно-фантастический формат притягивать к некому научному базису. Астрофизик-консультант - конечно интересно, но наебать так можно людей недалеких, которые и составят основную кассу. Не надо делать вид, что фильм несет за собой какие-то просветительские цели. Каким бы творцом не был режиссер, бюджет в сотни миллионов долларов надо отбить, потому что деньги - чужие. Не отобьет - потеряет доверие, и творить больше не дадут.
"Гравитация", "Аполлон 13" - вот тебе примеры фильмов не без косяков, но которые не прыгают за грань. В гравитации вообще ляпов - воз, но фильм хотя бы умещается в границах нынешней физики. "Интерстеллар" же напоминает мне "Фонтан" Аронофскикоторый лично мне очень даже понравился. Такая себе красивая философская сказка, заставляет задуматься! Зачем вообще таким вещам притягивать за уши какие-то научные стандарты.
>Не верти жопой.
И это лишнее. Цепляешься за мое мнимое незнание перевода "science fiction". Это чтоли нить разговора? Ну допустим не знаю я, и что тогда? +1 к "научности" фильма?
>>134759
>бро, ты по скриншоту умеешь определять расстояния?
Да, ты прав, двухмерность сыграла двумерно в моем сознании, обосрался.
>И вообще, фильм, может и говно
У меня нет никаких претензий к фильму и к Нолану в частности. Графон бесподобен, и, вроде как это первый высокобюджетный фильм, эксплуатирующий тематику на стыке теорий макромира и микромира. Возможно Нолан открыл путь к космической йобе без космических битв и сотен враждебных алиенов. Это охуенно. Просто не понравилось, что фильм, состоящий на 99% из домыслов и допущений называют научным.



Аноним 13/11/14 Чтв 12:46:16 #178 №134837 
14158719764820.png
>>134830
Абсолютно согласен.
Я бы уточнил что Гравитация и Аполло относятся скорее к жанру производственной драмы в космическом окружении. Как и Marooned. В их сценариях не требовалось особых научных допущении для связывания сюжета и создания декораций.
И я уверен что Нолан и команда никогда не ставили на "научность", не припомню такого в медиа.
Аноним 13/11/14 Чтв 12:53:11 #179 №134841 
>>134745
> Как будто аккреционный диск излучает исключительно в рентгене
Друк, этого рентгена более чем достаточно чтобы полностью стерилизовать все планеты в типовые Меркурии.
Аноним 13/11/14 Чтв 13:06:04 #180 №134848 
>>134841
>Продолжает читать жопой.
Молодец.
Аноним 13/11/14 Чтв 13:55:00 #181 №134854 
>>134830
> Не единой ОТО живет современная физика.

да, но тут всё-таки обсуждаются явления, которые она предсказывает и разрешает. А остальные теории никак их не подтверждают и не запрещают. Опять-таки кроме принципа причинности, который де-факто сейчас существует на странных правах вроде принципа абсолютности времени в эпоху классической физики. Всем будет больно, если найдутся нарушения, но это будет лютейший вин по глубине нашего незнания вселенной.

Не утверждается, что причинность есть всегда, или причинности нет, утверждается, что сейчас нет действующих теорий (многократно подтверждённых на практике, на основе которых работают какие-либо наши устройства), которые прямо запрещают эволюцию системы в обоих направлениях во времени. Даже та же стат.физика / термодинамика говорит лишь о крайне малой вероятности такого процесса.

> Просто не понравилось, что фильм, состоящий на 99% из домыслов и допущений называют научным

Здесь по некоторому размышлению соглашусь. Термин science-fiction, пожалуй, более удачен для обозначения класса подобных произведений. Научно-популярный жанр подразумевает отсутствие художественного вымысла, как пресловутая "Краткая история времени", например.


Ну и, кстати, не исключено, что причинность в понимании локальности взаимодействий на микроуровне может нарушаться - запутанное состояние частиц, квантовая телепортация и т.п.

https://ru.wikipedia.org/wiki/%D0%9D%D0%B5%D1%80%D0%B0%D0%B2%D0%B5%D0%BD%D1%81%D1%82%D0%B2%D0%B0_%D0%91%D0%B5%D0%BB%D0%BB%D0%B0#.D0.9D.D0.B0.D1.80.D1.83.D1.88.D0.B5.D0.BD.D0.B8.D0.B5_.D0.BF.D1.80.D0.B8.D0.BD.D1.86.D0.B8.D0.BF.D0.B0_.D0.BB.D0.BE.D0.BA.D0.B0.D0.BB.D1.8C.D0.BD.D0.BE.D0.B3.D0.BE_.D1.80.D0.B5.D0.B0.D0.BB.D0.B8.D0.B7.D0.BC.D0.B0_.D0.B8_.D1.81.D0.B2.D0.BE.D0.B1.D0.BE.D0.B4.D1.8B_.D0.B2.D1.8B.D0.B1.D0.BE.D1.80.D0.B0_.D0.B2_.D0.BE.D0.BF.D1.8B.D1.82.D0.B0.D1.85_.D0.A8.D0.B0.D0.B9.D0.B4.D0.BB.D0.B0_.D0.B8_.D0.B4.D1.80

- современные опыты говорят, что вполне вероятно, что взаимодействия в квантовом мире происходят быстрее скорости света.
Если всё это так, то нас в далёком или не очень будущем ждёт очередная революция в физике представлений о пространстве, времени и материи.
Аноним 13/11/14 Чтв 15:12:19 #182 №134866 
Понятно, что фильм абсолютно не научный!
Аргументы:
1.Ты дебил.
2.Нолан пидр.
3.Торн еврей.
Аноним 13/11/14 Чтв 15:21:00 #183 №134869 
>>134866
1. Нет ты
2, 3. Правда.
Аноним 13/11/14 Чтв 18:14:05 #184 №134896 
>>133400
Так, ладно. По расчётам, точка, в которой бы время замедлялось в 61320 раз, при гравитационном радиусе Гаргантюа, должна находится на расстоянии восьмидесяти метров от ГС. Это ноланофизика о двух концах: либо центр первой планеты удалён от ГС на расстояние 80 м плюс радиус самой первой планеты, либо он удалён на 80 м просто. В первом случае на высоте восьмидесяти метров над поверхностью, над головой Миклухи, находился ГС. Вероятно время замедляется из-за нихуёвой скорости вращения самой планетки вокруг ЧД, но из дегенератских объяснений команды Эндуранса понятно, что время замедляется только из-за влияния ЧД.
Аноним 13/11/14 Чтв 18:17:36 #185 №134897 
>>134896
респект за расчёты, неленивым везде у нас почёт. А можно расчёты выложить куда-нибудь? Интересно глянуть.
Аноним 13/11/14 Чтв 18:25:07 #186 №134900 
14158923071130.jpg
14158923071301.jpg
14158923071472.jpg
14158923071673.jpg
>>134897
Камера - говно.
Аноним 13/11/14 Чтв 21:47:13 #187 №134968 
14159044334890.jpg
>>134705
Почему не сможет?
Вот он захочет, попытается отдернуть руку, но какие-то неведомые силы будут управлят ьего рукой, как в Evil Dead и будут передавать координаты? Бред.
А если он даже не задумается об этом, значит вся наша жизнь предначертана? Т.е. можно и будущее предсказать. Ведь все что мы сделаем в будущем, по такой логике, имеет лишь 1 вариант. Бред.
>>134718
Вы договаривайте. Почему нельзя? Да потому что в прошлое нельзя попасть. Поэтому и изменить нельзя. Тот же колясочник, как я помню, отрицает возможность путешествия в прошлое.


Алсо пацаны, че вы за научность так заспорили. Всем очевидно, что он ненаучный, но никто и не говорит, что он ДОЛЖЕН быть научным. Я вообще написал "понятно что он не научный" в ОП-посте, лишь потому, что я тут впервые и подумал, что у вас все строго и меня сразу засагают, если подумают ,что мое мировозрение в какой-т омере соответствует картинке в фильме.

>>134896
Ты действительно крут!
Аноним 13/11/14 Чтв 22:03:07 #188 №134970 
>>133400
> Может ли наблюдатель выжить упав в черную дыру за горизонт событий?
Нет, "ниточное выдёргивание" же.
sageАноним 13/11/14 Чтв 22:06:31 #189 №134971 
Посоны, обсуждайте кино в отдельном, специальном треде, зачем вы на каждый фильм создаете отдельный тред, у нас тут не /tv/
Аноним 13/11/14 Чтв 22:12:27 #190 №134972 
>>134971
>у нас тут не /tv/
Нихуя ты отстал от жизни. Для фильмов отдельная доска два года уже как. И тебе что, жалко штоле? И так на нулевой висят треды годовой давности.
sageАноним 13/11/14 Чтв 22:28:11 #191 №134973 
>>134972
Вот и пиздуйте на отдельную доску.
Аноним 13/11/14 Чтв 22:29:28 #192 №134974 
>>134972
Я не он, но кино и правда лучше бы в один тред, тем более что фильмов про космос мало.
Аноним 14/11/14 Птн 02:51:09 #193 №135015 
>>134736

>Стоило Нолану ступить на путь собственного виденья, как фильм сразу перестал быть научным.

Когда он им был, вот в чем вопрос?

>>134743

Там рассуждения в стиле, что Кубрик был лучше и хуй стоял в 68ом. А Нолан говно, вот!

Аноним 14/11/14 Птн 11:33:58 #194 №135071 
>>134968
>Почему не сможет?
Выше ты согласился,что путешествие в будущее возможно.Значит оно - "есть".Относительно будущего Миклуха в прошлом.Прошлое изменить не можно.
Логика,диалектика.
>Бред.
Наивно верить в швободу воли.
Аноним 14/11/14 Птн 11:48:00 #195 №135079 
14159548801570.jpg
>>135071
Ну вот по мне, будущее есть, да.
А путешествие в прошлое невоможно, потому что рождает противоречие, которое невозможно разрешить, кроме как не настраивая еще дополнений, в стиле "параллельние вселенные" или классическое "Он не посмеет!!!".

Ну почему же наивно? Я же действительно могу сейчас написать тебе, а могу забить хуй и не написать. С другой стороны, в любом случае, я сделаю лишь что-то одно и никогда не совершу что-то другое (путешествие в прошлое невозможно). Так что всегда будет существовать лишь один вариант. Но мы же с тобой понимаем, что это уже философия и не больше...
Аноним 14/11/14 Птн 12:46:22 #196 №135096 
>>135079
>Я же действительно могу сейчас написать тебе, а могу забить хуй и не написать.
Нет.
если
>будущее есть, да.
то
Выбора нет.
иначе
(выбор есть)
Будущего нет.

>"параллельние вселенные"
Твоих рук дело.
Если есть выбор и есть будущее,то это будущее есть для каждого варианта выбора.

Аноним 14/11/14 Птн 13:00:45 #197 №135103 
14159592454890.jpg
>>135096
Будущее не в плане действий определенных, а в плане того, что в одних условиях распад цезия будет происходит быстрее, нежели в других условиях. неправильно выразился.

Теперь напиши свое мнение заново, плиз, насчет
>>вселенные" или классическое "Он не посмеет!!!"/ Ну почему же наивно? Я же действительно могу сейчас написать тебе, а могу забить хуй и не написать. С другой стороны, в любом случае, я сделаю лишь что-то одно и никогда не совершу что-то другое (путешествие в прошлое невозможно). Так что всегда будет существовать лишь один вариант. Но мы же с тобой понимаем, что это уже философия и не больше...

Учитывая мою поправку
Аноним 14/11/14 Птн 14:08:42 #198 №135114 
>>135103
>Теперь напиши свое мнение заново, плиз.
Мнение
Ты инфантилен.Вместо того,что бы принять критику,и понять ошибочность своей теории,что выгодно в первую очередь для тебя,ты пытаешься делать смысловые кульбиты.Получается плохо.Есть большая вероятность насрать себе за шиворот.
Аноним 14/11/14 Птн 14:40:45 #199 №135120 
>>135114
Инфантилен, да, возможно. Но я не пытаюсь вывернуться,а действительно высказываю свое мнение! Я не стараюсь тебя в чем-то убедить. Просто мне интересно тебя послушать.
Аноним 14/11/14 Птн 20:02:22 #200 №135217 
>>135114
Так вышло,что для мышления я использую логику.Теория причинно-следственной связи меня вполне устраивает.Я не отрицаю наличие более фундаментального закона/теории. Я даже поверхностно понимаю,что пытаются донести до нас адепты м-теории,но
>действительно высказываю свое мнение!
Перечитав твои посты итт,я не могу понять способ твоего мышления.Более того.
Я,ни хуя не могу понять "твое мнение".
Если в рамках школьной арифметики,твоё мнение 2+2=5,то на хуй такое мнение.

Вот тут:
>Просто мне интересно тебя послушать.

>Будущее не в плане действий определенных
>Я же действительно могу сейчас написать тебе, а могу забить хуй и не написать.
>не в плане действий определенных действий
>а в плане того, что в одних условиях распад цезия...
мой логический аппарат достиг предельной массы,и обвалившись внутрь себя,достиг сингулярности.Просветление.

Впрочем,ты можешь упрекнуть меня в знании рузке языка,и будешь абсолютно прав.



Аноним 14/11/14 Птн 20:10:38 #201 №135220 
14159850386070.jpg
>>135217
Мне просто было интересно послушать мнения людей на то, что бы произошло, если бы он не передал координаты, но мнения "он бы не посмел" или "ему бы помешала какая-либо сверхсила" меня не удовлетворили. Вот и все.
Аноним 15/11/14 Суб 03:00:58 #202 №135312 
>>135220
>что бы произошло, если бы камень упал вверх.
А хуй его знает :3
Аноним 15/11/14 Суб 05:24:26 #203 №135323 
>>135312
Чтобы нп задавать вопросов из разряда "что бы произошло, если бы камень упал вверх?" жля начала надоотказаться от утверждения, что гравитация Знмли отталкивает тела от нее
Аноним 15/11/14 Суб 07:56:38 #204 №135331 
>>135323
>жля
>надоотказаться
>гравитация Знмли отталкивает тела от нее
Шизик, please.
Аноним 15/11/14 Суб 08:41:50 #205 №135334 
14160301105840.jpg
>>135331
Да я с телефона просто. Суть понятна, надеюсь
Аноним 15/11/14 Суб 08:51:04 #206 №135335 
>>135334
>Суть понятна
Суть в том, что ты глуп.
Это, разумеется, понятно.
Аноним 15/11/14 Суб 10:01:02 #207 №135340 
>>135220
>Мне просто было интересно послушать мнения людей
Нет,бро."Лалки/уебаны/пидоры" показывают,что на мнение людей тебе похуй.Ты даже не можешь понять,что это не "мнения",а выводы при помощи общепринятой(какбы допускаю возможность иной) логики.Но это не самое хуёвое.Самый пиздец в том,что вас дохуя ирл.И вы не хуёво,так,заняли место в обществе.Вы управляете автомобилями,занимаете должности,имеете семью.
Мне, интересен способ твоего мышления.Я уверен ,что он(способ)-есть.Ведь вы коммуницируете,особенно друг с другом.
Вы-альтернативно мыслящие,лекго приходите к консенсусу.Например:надо тыкать вилкой в шею потому,что чайник красный.И не доебёшся же,до такой теории. Не надо тыкать вилкой в шею потому,что чайник красный?Надо тыкать вилкой не в шею?Надо тыкать не вилкой?
Запрашиваю способ мышления.
твой альтернативно грамотный друк
Аноним 15/11/14 Суб 10:07:15 #208 №135341 
14160352350230.jpg
Да че вас заносит т всех куда-то в дали дальние.
Есть конкретный вопрос. Почему начинается обсуждение лично меня?

"...что бы произошло, если бы он не передал координаты, но мнения "он бы не посмел" или "ему бы помешала какая-либо сверхсила" меня не удовлетвор(яют). Вот и все. "
Аноним 15/11/14 Суб 10:24:53 #209 №135344 
14160362937800.jpg
Алсо я только сейчас понял, что тебя тае смущало в путешествиях в будущее. Невнисательно читал.
Я писал, что допкскаю возможность путешествия в будущее, но нигде не говорится, что следствием этого является существование уже в данный момент какого-то определённого будущего. Тут становится понятно про цезий. Путешествие в будущее, которо я имел ввиду, не есть телепортация блеать, просто пока у одного наблюдателя проходит одно количество "времени", у другого проходит другое. Поэтомк нет никакого будущего определенного,относительно котороо Миклуха был бы в прошлом. Поэтому и " выбор"есть. Хз, поправьте мою поехавшую логику, если есть желание.

Во всяком случае этот пост нарисан К СЛОВУ. А основноц вопрос, как и прежде, в предыдущем

Аноним 15/11/14 Суб 10:24:57 #210 №135345 
>>135341
>обсуждение лично меня?
Потому что вопрос глуп на грани кретинизма легкой степени - как и ты сам, впрочем.
То есть в этом смысле оба предмета обсуждения равноправны - но тут есть еще и этологический интерес. Зоонаблюдения же - не каждый день такое как ты забегает.
Так-то.
Аноним 15/11/14 Суб 10:36:20 #211 №135346 
>>135345
Я обещаю тебе, что отвечу тебе на любой вопрос относительно меня, чтобы ты продолжил свои наблюдения, но ответь мне

"...что бы произошло, если бы он не передал координаты, но мнения "он бы не посмел" или "ему бы помешала какая-либо сверхсила" меня не удовлетвор(яют). Вот и все. "
Аноним 15/11/14 Суб 10:43:30 #212 №135348 
>>135341
>"...что бы произошло, если бы он не передал координаты, но мнения "он бы не посмел" или "ему бы помешала какая-либо сверхсила" меня не удовлетвор(яют). Вот и все. "
Я, вот здесь >>135096 тебе всё разжевал.

Впрочем,я все понял.Ты женщина.Хули я тут к логике ссылаюсь?
Вот,как то так,тогда:
Кароч ,он такой захател,и при помощи силы воли взял и не передал координаты.Не посмеет?-ха ха!Сверхсила?-хахаха).МаТиМаТиКа?-лол!ЛоГиКа?-узБагОЙСЯ 8).
по прежнему твой

Аноним 15/11/14 Суб 10:52:48 #213 №135349 
>>135346>>135348
Борьба была равна, боролись два говна.
^_^
Аноним 15/11/14 Суб 10:57:05 #214 №135350 
14160382251460.jpg
>>135348
... Окей. Спасибо за дискуссию.)
Алсо нет, не женщина

Есть кто-нибудь, кто может высказаться кроме данного господина?
Аноним 15/11/14 Суб 10:58:14 #215 №135351 
>>135350
>нет, не женщина
Разумеется. Простое питурдообразное, без пола но с гормональными нарушениями.
Аноним 15/11/14 Суб 12:46:34 #216 №135366 
>>135344
>Я писал, что допкскаю возможность путешествия в будущее
>нет никакого будущего определённого будущего.
Ну и куда ты тогда пропутешествуешь?
>допкскаю возможность путешествия в будущее
Подразумевает "наличее" будущего.

Ладно,я,наверное, смутно понял твои жизафренические калькуляции.

Подумай вот о чём:
Если для двух разных наблюдателей,"фактически",время течет с различной "скоростью"(но,для них с одинаковой),что мы можем принять за "скорость времени"? Только твоё сознание?
всё еще твой безграмотный котик
Аноним 15/11/14 Суб 13:29:50 #217 №135371 
14160473902650.jpg
>>135366
Куда пропутешествую? А куда ты пропутушествуешь, открыв завтра утром глаза? Относительно своей "точки отсчета" - да никуда. Тут надо сравнивать с чем-то (читай дальше)

Да, ты правильно понял. Не подразумевает наличие будущего, потому что просто для двух наблюдателей время течет относительно друг друга по-разному.

>> Путешествие в будущее, которо я имел ввиду, не есть телепортация блеать, просто пока у одного наблюдателя проходит одно количество "времени", у другого проходит другое. Поэтомк нет никакого будущего определенного,относительно котороо Миклуха был бы в прошлом


Насчет того, что принять за скорость времени... Да то же самое, что и сейчас - распад определённого элемента. По факту " для них" по отдельности, как ты и написал, время будет течь одинаково, но относительно друг друга будут изменения. Вот у кого быстрее текло, тот и оказался в будущем, относительно второго. Как-то так.

Если не понятно ты проси расписать. Мои мысли спутаны и затемнены, но мне нравится что мы вернулись к нормальному диалогу


Аноним 15/11/14 Суб 13:58:44 #218 №135375 
14160491241180.jpg
К СЛОВУ
Мне кажется такой взгляд как раз таки менее шизовый, чем мнение с биологическими безвольными машинами, определенным будущем и прочим фатализмом.)
На противоречия в логике указывай мне, если я запёздываться начну
Аноним 15/11/14 Суб 16:55:49 #219 №135410 
117048
Аноним 15/11/14 Суб 17:08:35 #220 №135418 
14160605153920.jpg
На каждой доске, блять, этот ваш Интерстеллар. В сайфаче, в быдлаче, везде!
sageАноним 15/11/14 Суб 17:47:29 #221 №135427 
>>135418
Умерьте свой бушприт, сэр. Если фильм так активно обсуждают, значит есть что обсуждать.
Аноним 15/11/14 Суб 18:22:45 #222 №135430 
>>135418
Перестанут платить за пиар этого говна - пропадут треды, посвященные этому говну.
sageАноним 15/11/14 Суб 18:24:56 #223 №135431 
>>135418
Просто фантастика нынче редка.
sageАноним 15/11/14 Суб 20:51:24 #224 №135457 
>>135430
Дааа, конечно, ананасам с двача платят за пиар Итерстеллара, ты совсем ебанулся что ли?
sageАноним 15/11/14 Суб 23:43:17 #225 №135509 
>>135457
Сотни тредов в /б/ с кучей одинаковых постов БРАВО НОЛАН - нет, это не проплаченный пиар, конечно.
И ОП-хуй создал тред здесь не просто так. И в /мов/ это говно натужно расхваливают не за так.
Аноним 15/11/14 Суб 23:53:43 #226 №135519 
>>135509
Кстати, ОПу фильм не особо понравился. Но так как Нолан и Маклуха, то сходить надо.
Оп-хуй
sageАноним 16/11/14 Вск 01:05:04 #227 №135546 
>>135509
Трудновато тебе живется. Все всё всегда делают за деньги и все кругом проплачено. Нда. Слушай, а ты к врачам не пробовал с этой проблемой обращаться?
sageАноним 16/11/14 Вск 11:40:37 #228 №135618 
>>135546
Не прибедняйся. поди ты и создавал тысячи тредов в /б/ с пиаром этого говна. Каково это - наебывать за жалкие копейки себе подобных?
Это из-за тебе подобных человек не ступал на Марс и не ковырял льды Европы - ненавижу ебаное общество потребления, ебаных ноланов и ебаного тебя, за копейки пиарящего говно, которое ноланы призводят.
Аноним 16/11/14 Вск 13:27:06 #229 №135630 
>>135366
>что мы можем принять за "скорость времени"?
По идее нам нужен некоторый универсальный источник времени, лежащий за пределами нашей галактики, а еще лучше за пределами нашей Вселенной. Ибо наша галактика, сцуко, тяжёлая и время на сколько-нибудь она замедляет для всех объектов лежащих как внутри неё, так и в непосредственной близости. Но т.к. таких универсальных часов мы не имеем, то остаётся сравнивать лишь относительную скорость течения времени для двух объектов, имеющих разную массу и/или скорость.
Аноним 16/11/14 Вск 17:23:20 #230 №135683 
>>133495
> Джетов, излучения. Просто поджарит издалека.
Вот это вот не очень понятно. Если даже фотон не может преодолеть гравитацию чёрной дыры, то как она может излучать?
sageАноним 16/11/14 Вск 17:45:43 #231 №135691 
>>135683
Излучение из черной дыры только планковское, а сама дыра светится в разных диапазонах из-за сжатия газов до горизонта событий.
Аноним 16/11/14 Вск 18:20:29 #232 №135701 
>>135630
>некоторый универсальный источник времени
Я таки извиняюсь, но это напоминает об универсальной системе отсчёта. Ща кефирщики набигут.
sageАноним 20/11/14 Чтв 04:48:56 #233 №136525 
Таки извиняюсь, но кино лучше обсуждать в едином кинотреде.

https://2ch.hk/spc/res/77836.html
comments powered by Disqus

Отзывы и предложения